Download 2009 EDX Knowledge Assessment Questions

Document related concepts

Nerve guidance conduit wikipedia , lookup

Auditory brainstem response wikipedia , lookup

Auditory system wikipedia , lookup

Smile surgery wikipedia , lookup

Transcript
2009 EDX SAE QUESTIONS
51.
Which finding listed is the best way to separate a medial cord from a lower trunk plexopathy?
A.
B.
C.
D.
E.
52.
When compared to large diameter axons, small diameter axons demonstrate:
A.
B.
C.
D.
E.
53.
Increased axoplasmic resistance.
Thicker myelin.
Longer internodal distances.
Greater blood flow.
Faster conduction velocity.
The A wave, also referred to as the axon reflex, is characterized by which of the following features?
A.
B.
C.
D.
E.
54.
Low ulnar sensory amplitude.
Fibrillation potentials in extensor indicis proprius.
Low median motor amplitude to the abductor pollicis brevis.
Fibrillation potentials in abductor pollicis brevis.
Low median sensory amplitude to the index finger.
Onset before the M wave.
Variable onset latency.
Increasing latencies with more proximal stimulation.
A constant wave form.
It occurs with supramaximal stimulation.
Which statement about the median nerve is true?
A. Routine median motor nerve conduction studies are abnormal in upper trunk brachial
plexopathies.
B. Median sensory nerve conduction studies to the index finger are typically normal in lower trunk
brachial plexopathies.
C. The anterior interosseous nerve branches proximal to the pronator teres muscle.
D. The nerve can be compressed at the Arcade of Frohse.
E. The nerve is commonly affected in posterior cord brachial plexopathies.
55.
Failure to adequately warm a limb prior to performing nerve conduction studies may result in all of
the following EXCEPT:
A.
B.
C.
D.
E.
20
Decreased amplitude of the sensory nerve action potential.
Prolonged distal latency.
Slowing of conduction velocity.
Increased duration of the action potential.
Erroneous results on repetitive stimulation.
2009 AANEM Electrodiagnostic Self-Assessment Examination
56.
Slow repetitive stimulation studies in myasthenia gravis:
A.
B.
C.
D.
E.
57.
Motor nerve conduction velocities reach adult values at what age?
A.
B.
C.
D.
E.
58.
Abundant fibrillations and positive sharp waves.
Decreased motor unit action potential recruitment.
Early motor unit action potential recruitment.
Increased motor unit action potential amplitude.
Increased motor unit action potential duration.
In a patient who underwent mastectomy and postoperative radiation therapy for breast cancer and
later developed arm weakness, which of the following would best support that her weakness is due
to radiation plexopathy rather than tumor recurrence?
A.
B.
C.
D.
E.
60.
6 months to 1 year.
1 year to 1.5 years.
1.5 years to 2.5 years.
3 years to 5 years.
6 years to 8 years.
In an acute radiculopathy (first 7 days), which abnormal finding is the most likely to be seen on
needle electromyography of an affected muscle?
A.
B.
C.
D.
E.
59.
Typically show a larger decrement if the limb is cool.
Typically show the greatest decrement from the first to the second response.
Are not influenced by anticholinesterase drugs.
Frequently show greater than 100% facilitation by the third response.
May be considered abnormal with a 5% decrement response.
Myokymic discharges.
Fibrillation potentials.
Absent ulnar sensory nerve action potential (SNAP).
Absent medial antebrachial cutaneous SNAP.
Pain as a predominant symptom.
What is the advantage of a constant current stimulator versus a constant voltage stimulator when
performing nerve conduction studies?
A. Constant current stimulators deliver a consistent stimulus current that is impedance
dependent.
B. Constant voltage stimulators deliver consistent current stimulation that is impedance
independent.
C. Constant current stimulators deliver a consistent current that is impedance independent.
D. Constant voltage stimulators deliver a varying current stimulus that is impedance dependent.
E. There is no advantage of one system over the other as both deliver a stimulus current.
2009 AANEM Electrodiagnostic Self-Assessment Examination
21
61.
Perifascicular atrophy on muscle biopsy is most characteristic of:
A.
B.
C.
D.
E.
62.
Which is a recognized site of distal ulnar nerve entrapment?
A.
B.
C.
D.
E.
63.
Ligament of Struthers.
Lacertus fibrosis.
Pronator teres muscle.
Brachialis muscle.
Sublimis bridge.
A patient with persistent right hand and forearm pain is referred to you for electrodiagnostic studies
by a hand surgeon; his diagnosis is persistent carpal tunnel syndrome (CTS). Carpal tunnel
release surgery 1 year ago by a different surgeon produced no benefit in symptoms. You find
severe axonal loss, but borderline distal motor latency of the right median motor and normal right
median sensory responses. What is the best next course of action?
A.
B.
C.
D.
E.
22
Fibrillations in some or all involved muscles.
Decreased sensory nerve action potential amplitudes.
Decreased compound muscle action potential amplitudes.
Increased polyphasic motor unit action potentials in some or all involved muscles.
Increased distal motor latency.
Proximal median mononeuropathy occurs at all sites listed EXCEPT:
A.
B.
C.
D.
E.
65.
Guyon’s canal.
Ligament of Struthers.
Arcade of Frohse.
Spiral groove.
Transcarpal ligament.
The first change to be seen in a severe axon-loss plexopathy is:
A.
B.
C.
D.
E.
64.
Duchenne muscular dystrophy.
Fascioscapulohumeral dystrophy.
Inclusion body myositis.
Polymyositis.
Dermatomyositis.
Perform a more detailed right and limited left arm study to better localize the process.
Inform the patient that they do not have CTS and that the prior surgery was unnecessary.
Contact the referring physician to ask permission to perform a more complete arm study.
Perform a 3-limb comprehensive study to screen for a generalized, but unsuspected process.
Conclude that the patient has severe CTS and notify the surgeon that the patient needs repeat
surgery.
2009 AANEM Electrodiagnostic Self-Assessment Examination
66.
A 44-year-old male is referred for evaluation of a left lumbosacral radiculopathy. The tibial H-reflex
latencies are 31.5 ms (left) and 31.6 ms (right). The antidromic sural sensory peak latency at 14 cm
is 3.8 ms with an amplitude of 12 µV. The left peroneal motor nerve distal latency is 4.3 ms with an
amplitude of 1.8 mV and with stimulation at the knee, the conduction velocity is 44 m/s with an
amplitude of 3.5 mV. Tibial motor responses are normal. Which additional study would be most
helpful?
A. Peroneal motor conduction study on the opposite side.
B. Stimulation posterior to the lateral malleolus while recording from the extensor digitorum brevis
muscle.
C. Peroneal motor responses to the peroneus longus muscle.
D. Motor and sensory nerve conduction studies in an upper extremity.
E. Peroneal sensory conduction study on the left side.
67.
Which of the following choices is true regarding the musculocutaneous nerve?
A.
B.
C.
D.
E.
68.
Which of the following is true regarding an abnormally complex motor unit?
A.
B.
C.
D.
E.
69.
Phases and turns are not a factor.
It has four phases or four turns.
It has four phases or five turns.
It has more than four phases or more than four turns.
It has more than four phases or more than five turns.
In a median to ulnar crossover anomaly in the forearm, which finding is most likely observed?
A.
B.
C.
D.
E.
70.
It originates from the posterior cord of the brachial plexus.
It provides motor fibers to the biceps, brachialis, and brachioradialis muscles.
Its terminal sensory branch supplies sensation to the medial forearm.
It may communicate with the median nerve in the arm.
In distal musculocutaneous neuropathies at the elbow, needle electromyography is abnormal.
False median nerve conduction block in the forearm recording from the abductor pollicis brevis.
Slowing of ulnar conduction velocity in the forearm.
False ulnar nerve conduction block in the forearm recording from the abductor digiti minimi.
Slowing of median conduction velocity in the forearm.
Low ulnar sensory nerve amplitude.
Which of the following is a near-field somatosensory evoked potential?
A.
B.
C.
D.
E.
N13.
P14.
N18.
P31.
N34.
2009 AANEM Electrodiagnostic Self-Assessment Examination
23
71.
Which activity listed below characteristically starts and stops abruptly?
A.
B.
C.
D.
E.
72.
Entrapment of the suprascapular nerve at what level causes isolated atrophy of the infraspinatus
muscle?
A.
B.
C.
D.
E.
73.
Biceps brachii; anconeus.
Brachioradialis; triceps.
Abductor pollicis longus; pronator quadrates.
Extensor digitorum communis; pronator teres.
Flexor carpi radialis; brachioradialis.
In Lambert-Eaton myasthenic syndrome:
A.
B.
C.
D.
E.
24
2 Hz.
10 Hz.
18 Hz.
20 Hz.
22 Hz.
The muscle pair most likely to receive a significant portion of its innervation from C7 is:
A.
B.
C.
D.
E.
75.
The suprascapular notch.
The spinoglenoid notch.
The quadrangular space.
The interscalene triangle.
Erb’s point.
While performing a needle electromyography study with the sweep speed set at 10 ms/division and
10 divisions on the screen, one voluntary motor unit action potential is seen twice on each sweep
and appears to be moving slowly towards the right. What is the frequency of the potential?
A.
B.
C.
D.
E.
74.
Fibrillation potentials.
Positive sharp waves.
Complex repetitive discharges.
Myotonic discharges.
Myokymic discharges.
Antibodies attack postsynaptic voltage-gated calcium channels.
Antibodies attack presynaptic voltage-gated sodium channels.
Antibodies attack presynaptic voltage-gated calcium channels.
Sodium influx during depolarization in impaired, inhibiting release of acetylcholine.
Sodium uptake during repolarization is impaired.
2009 AANEM Electrodiagnostic Self-Assessment Examination
76.
To reduce electrical noise, some electrodiagnostic laboratories are supplied with a dedicated
electrical power circuit and isolated ground. This reduces noise by:
A.
B.
C.
D.
E.
77.
The most sensitive single test of impaired neuromuscular transmission in myasthenia gravis is:
A.
B.
C.
D.
E.
78.
Decremental response on repetitive nerve stimulation of a distal limb muscle at 10 Hz.
Decremental response on repetitive nerve stimulation of a proximal limb muscle at 3 Hz.
Increased blocking during single fiber electromyography (SFEMG) of a forearm muscle.
Increased jitter during SFEMG of the frontalis muscle.
Elevated acetylcholine receptor antibody titers.
You examine a 32-year-old-patient with a clinical history of multiple entrapment neuropathies at
different time points. Electrodiagnostic findings confirm focal lesions at typical entrapment sites in
multiple nerves. Which laboratory abnormality is most likely to be found in this patient?
A.
B.
C.
D.
E.
79.
Shielding the room from extraneous radiated noise sources.
Minimizing noise added to the power line from other equipment.
Enhancing the current carrying capacity of the ground.
Using special noise reducing circuitry that conditions the power lines.
Eliminating 60 Hz noise from the power lines.
A mutation of the transthyretin gene.
A deletion of the PMP-22 gene.
Elevated methylmalonic acid level.
Elevated GM1-ganglioside antibody titer.
Chest computed tomography demonstrating small cell lung cancer.
Short-duration, low-amplitude polyphasic motor unit action potentials are most likely associated
with:
A.
B.
C.
D.
E.
Duchenne muscular dystrophy.
Old poliomyelitis.
Adult-onset spinal muscular atrophy.
Acute (2 weeks duration) radiculopathy.
Chronic inflammatory demyelinating polyneuropathy.
2009 AANEM Electrodiagnostic Self-Assessment Examination
25
80.
32-year-old female with a history of ptosis is referred for repetitive nerve stimulation studies.
Stimulation of the right median nerve at 2 Hz is shown in the Figure above. Which of the following
is the most likely explanation?
A.
B.
C.
D.
E.
81.
A 29-year-old woman, complaining of weakness that appears to be functional, is seen in the
electromyography (EMG) laboratory. Nerve conduction studies of the ulnar and median nerves are
normal. The needle EMG is normal except for excessive variation in amplitude of some of the
individual motor unit action potentials. What additional studies are indicated?
A.
B.
C.
D.
E.
26
Abnormal decremental response of 19%.
Lowering of the intramuscular temperature from 32ºC to 28ºC.
Supramaximal stimulation.
Movement induced artifact.
Normal response.
None, the study supports a functional disorder.
Repetitive nerve stimulation studies or single fiber EMG.
Proximal conduction studies in the upper extremities.
Repeat EMG studies with an increased low frequency filter setting.
Repeat EMG after muscle warming.
2009 AANEM Electrodiagnostic Self-Assessment Examination
82.
A 32-year-old female is evaluated for increasing hand numbness, muscle cramps, and proximal
muscle weakness for about 2 years. She noted increasing fatigue, weight gain, and hair loss.
Family history is negative for neuromuscular disorders.
On examination, she has diffusely hypoactive reflexes and positive Tinel sign at the wrists, and she
reports decreased pin-prick sensation in the fingertips and in the toes. She has moderate bilateral
hip flexor weakness, mild deltoid weakness, and no muscle atrophy. Nerve conduction velocity
shows moderately prolonged distal motor latency and absent sensory amplitudes in the bilateral
median nerves, and low amplitude sural potentials in both legs. The other nerves tested are all
normal. Needle electromyography shows a few positive sharp waves with small polyphasic motor
units in the proximal arm and leg muscles.
Which of the following would you recommend as the appropriate next step:
A.
B.
C.
D.
E.
83.
Which of the following statements characterizes the blink reflex?
A.
B.
C.
D.
E.
84.
The R1 component is recorded bilaterally with unilateral stimulation.
The afferent and efferent reflex arcs run in the same nerve.
It is a monosynaptic reflex.
It is initiated by antidromic stimulation of motor fibers.
The R2 component follows a polysynaptic pathway through the pons and lateral medulla.
Which finding listed is the earliest electromyographic change seen in acute radiculopathies with
axonal loss?
A.
B.
C.
D.
E.
85.
Serum levels of thyroid stimulating hormone and free T4.
Serum creatine kinase and cortisol level.
Repetitive nerve stimulation of a distal and proximal system.
Quadriceps muscle biopsy.
Cervical and lumbar spine magnetic resonance imaging survey.
Fibrillation potentials or positive sharp waves in paraspinal muscles.
Fibrillation potentials or positive sharp waves in proximal limb muscles.
Fibrillation potentials or positive sharp waves in distal limb muscles.
Increased number of polyphasic motor unit action potentials (MUAPs) in paraspinal muscles.
Increased number of polyphasic MUAPs in distal limb muscles.
In a lesion affecting the lumbar plexus, which muscle pair would most likely show abnormal
findings on needle electromyography?
A.
B.
C.
D.
E.
Rectus femoris; gracilis.
Vastus medialis; semimembranosus.
Gluteus maximus; adductor longus.
Adductor magnus; piriformis.
Peroneus tertius; vastus lateralis.
2009 AANEM Electrodiagnostic Self-Assessment Examination
27
86.
Following complete nerve transaction and repair, which one of these findings coincides with early
reinnervation?
A.
B.
C.
D.
E.
87.
A return of conduction velocity to normal.
The development of abundant fibrillations.
The disappearance of positive waves.
The development of low-amplitude, polyphasic motor unit action potentials (MUAPs).
The appearance of a few giant MUAPs.
A 36-year-old male fell off a roof 3 weeks ago, fracturing his pelvis including his right acetabulum.
Moving the right leg is still painful, but he also notes inability to dorsiflex the right ankle and
numbness of the dorsum of the right foot since the fall.
The right lower extremity nerve conduction study finding that would help to localize the lesion distal
to the root level is a/an:
A.
B.
C.
D.
Absent peroneal motor response from extensor digitorum brevis muscle.
Normal sural sensory distal latency.
Reduced superficial peroneal sensory action potential amplitude.
Reduced tibial motor compound muscle action potential amplitude from the abductor hallucis
muscle.
E. Absent H-reflex response.
88.
Motor evoked potential responses in hand muscles elicited by transcortical magnetic stimulation at
the scalp with a round coil centered at the vertex:
A. Show compound muscle action potentials with invariant onset latency and amplitude to
consecutive single stimuli delivered once per second.
B. Are significantly affected by preceding voluntary hand muscle contraction.
C. Are not mediated by indirect waves.
D. Are insensitive to the direction of monophasic induced current in the brain.
E. Show identical evoked amplitudes as distal nerve stimulation.
89.
An electrodiagnostic medicine physician inadvertently places the low-frequency filter at 500 Hz
when performing motor nerve conduction studies. As a result, the recorded action potentials will
have:
A.
B.
C.
D.
E.
28
An increase in amplitude.
An increase in duration.
A decrease in amplitude.
A decrease in the number of phases.
A positive initial deflection.
2009 AANEM Electrodiagnostic Self-Assessment Examination
90.
A 35-year-old man presents with foot drop after prolonged crossing of his legs during an airline
flight. You suspect a peroneal neuropathy. At what point in time after onset of weakness can the
motor nerve conduction studies of the peroneal nerve provide definitive information regarding
whether the foot drop is due to a severe axon loss or neurapraxia?
A.
B.
C.
D.
E.
91.
You are asked to perform a needle electromyography (EMG) study on an indigent patient in the
intensive care unit who was admitted with acute onset weakness, but is now showing minimal
improvement. What should you do?
A.
B.
C.
D.
E.
92.
Perform the study.
Defer the study because the patient is stable.
Refer the patient to the County hospital’s outpatient EMG laboratory.
Refuse, because the patient lacks financial resources to cover the study cost.
Inform the patient that the study is medically unnecessary.
A 26-year-old woman has had increasing trouble using her hands. Her medical history is
unremarkable except for bilateral cataract surgery at the age of 19. Examination demonstrates
bilateral distal weakness in all four extremities with slight weakness of the facial muscles and
moderate weakness and atrophy of the sternocleidomastoid muscles. Which test is most likely to
provide a diagnosis?
A.
B.
C.
D.
E.
93.
Within 24 hours.
1 to 3 days.
4 to 7 days.
10 to 14 days.
21 days.
Nerve conduction studies.
Needle electromyography.
Muscle biopsy.
Nerve biopsy.
Creatine kinase levels.
The findings of decreased abductor pollicis brevis and abductor digiti minimi compound muscle
action potential amplitude, abnormal ulnar sensory nerve action potential (SNAP) amplitude, and
normal median SNAP amplitude are most consistent with which of the following?
A.
B.
C.
D.
E.
Neurogenic thoracic outlet syndrome.
Ulnar nerve elbow segment neuropathy.
Ulnar compression at the elbow and median compression at the ligament of Struthers.
Lower and middle trunk brachial plexopathy.
C8 cervical radiculopathy.
2009 AANEM Electrodiagnostic Self-Assessment Examination
29
94.
An 18-year-old male struck a tree while sledding. He recalls striking the tree with his left shoulder.
Following the accident, he noted immediate shoulder pain, an inability to move his arm, and left
arm numbness. He was unable to voluntarily move his arm and he had decreased hand and
forearm sensation. Electrodiagnostic studies were performed 3 weeks following the injury. Nerve
conduction studies demonstrated absent left median and ulnar evoked motor responses. Needle
electromyography showed sustained positive waves and fibrillations in the intrinsic median and
ulnar hand muscles on the left. Which one of the following findings would best favor a lower trunk
brachial plexus lesion over a radiculopathy in the patient?
A.
B.
C.
D.
E.
95.
Motor neurons that innervate type I muscle fibers when compared to motor neurons that innervate
type II muscle fibers:
A.
B.
C.
D.
E.
96.
Have faster conduction velocity.
Have a lower threshold for recruitment.
Have a higher firing frequency.
Have a higher frequency of miniature end-plate potentials.
Have a larger axon diameter.
A patient undergoes an electrodiagnostic study, as part of the work up for arm pain. Nerve
conduction studies disclose normal ulnar motor (recording hypothenar eminence) and sensory
(recording the ring finger) studies. The median sensory (recording from the index finger) studies
are normal, but the median motor studies (recording from thenar eminence) reveals a very low
response with stimulation at the wrist and elbow. The median distal motor latency is normal. The
thenar eminence appears of normal bulk and power on clinical examination and the needle
electrode examination of the abductor pollicis brevis muscle is normal. Stimulation of the ulnar
nerve at the wrist while recording over the thenar eminence discloses a normal motor response.
This constellation of findings is most consistent with:
A.
B.
C.
D.
E.
30
A normal median F-response latency.
Absent ulnar F responses.
An absent DII median sensory response.
An absent ulnar sensory response.
A normal ulnar sensory response.
T1 motor radiculopathy.
Lower trunk brachial plexopathy.
Nerve lesion of the recurrent thenar branch of the median nerve.
Martin Gruber anastomosis.
Riche-Cannieu anomaly.
2009 AANEM Electrodiagnostic Self-Assessment Examination
97.
Lambert-Eaton myasthenic syndrome is characterized by:
A.
B.
C.
D.
E.
98.
Which muscle is frequently supplied by a single spinal root level?
A.
B.
C.
D.
E.
99.
Utilizes a polysynaptic spinal cord reflex arc.
Has a maximum amplitude with submaximal stimulation.
Has a variable waveform at low rates of stimulation.
Can be recorded from stimulation of any nerve/muscle.
Depends upon backfiring of motor neurons.
You perform electrodiagnostic studies on a 60-year-old man with asymmetric proximal leg and
forearm weakness, notably affecting finger flexor strength. Myopathic changes are evident on
needle electromyography in involved muscles. Which pathological finding is most likely to be found
on muscle biopsy?
A.
B.
C.
D.
E.
101.
The supraspinatus muscle.
The teres minor muscle.
The rhomboid major muscle.
The deltoid muscle.
The teres major muscle.
In contrast to the F wave, the H reflex:
A.
B.
C.
D.
E.
100.
Diminished release of acetylcholine with brief exercise.
Blockage of presynaptic voltage-gated calcium channels.
Frequent incidence of associated thymoma.
Prominent ophthalmoparesis.
Muscle specific kinase antibodies.
Patchy dystrophin staining in muscle membrane.
Renaut bodies.
Congophilic material within rimmed vacuoles.
Nemaline rods.
Signs of vasculitis in small blood vessels.
Sensory nerve action potential amplitude will be increased by:
A.
B.
C.
D.
E.
Increasing stimulation intensity beyond supramaximal.
Decreasing extremity temperature.
Increasing the distance between stimulating and recording electrodes.
Decreasing the high-frequency filter setting.
Increasing recording electrode impedance.
2009 AANEM Electrodiagnostic Self-Assessment Examination
31
102.
Miniature end-plate potentials:
A. Represent spontaneous depolarizations of the presynaptic nerve terminal.
B. Only occur following a depolarization of the presynaptic nerve terminal following a nerve action
potential.
C. Are produced by a single quantum of acetylcholine spontaneously released from the
presynaptic nerve terminal.
D. In Lambert-Eaton myasthenic syndrome are reduced in amplitude.
E. In botulism are increased in amplitude.
103.
Sciatic mononeuropathy is best associated with:
A.
B.
C.
D.
E.
104.
Posttetanic potentiation:
A.
B.
C.
D.
E.
105.
Complications of caesarean section surgery.
Complications of hip surgery.
Diabetic nerve infarction.
Tight belts around the waist.
Retroperitoneal hematoma.
Is due to a reduction in the immediately available store of acetylcholine.
Is due to a depletion of calcium stores in the nerve terminal.
Is due to enhanced release of acetylcholine.
Occurs only in presynaptic neuromuscular transmission disorders.
Occurs only in postsynaptic neuromuscular transmission disorders.
Which one of the following statements best describes differential amplifiers?
A.
B.
C.
D.
Input voltage to the amplifier equals output voltage.
Input current to the amplifier equals output current.
Input impedance is set as low as possible.
Output voltage is proportional to the active electrode voltage minus the reference electrode
voltage.
E. Input current to the amplifier is derived from the electromyography machine’s power supply,
which is connected to the wall current.
106.
After the age of 50 years, which of the following changes on motor nerve conduction studies is
observed?
A.
B.
C.
D.
E.
32
A slowing of conduction velocity of 3 to 4 m/s per decade.
A slowing of conduction velocity of 1 to 2 m/s per decade.
An increase in motor amplitude due to synchronization.
A reduction in distal motor latency.
Reduction in response duration.
2009 AANEM Electrodiagnostic Self-Assessment Examination
107.
A previously healthy 54-year-old male developed severe burning pain in his fingertips and
numbness in his feet with profound fatigue and mild cardiomyopathy over the last 4 months. He
suffers with impotence for 1 year and now has recurrent diarrhea. There is no history of diabetes,
normal fasting glucose, and no known toxic exposure. Nerve conduction testing shows absent
sensory potentials in the lower extremities and severe bilateral carpal tunnel syndrome. Which one
of the following tests will be most likely abnormal and diagnostic?
A.
B.
C.
D.
E.
108.
Compound muscle action potential amplitude will be increased by:
A.
B.
C.
D.
E.
109.
Increasing the stimulus intensity beyond supra-maximal.
Increasing extremity temperature.
Decreasing the low frequency filter.
Increasing the distance between the recording and stimulating electrodes.
Increasing the recording electrode impedance.
A characteristic of motor end-plate potentials recorded during needle electromyography is:
A.
B.
C.
D.
E.
110.
Serum levels of vitamin B12 and methylmalonic acid.
Glucose tolerance test.
Cerebral spinal fluid Lyme polymerase chain reaction.
Rectal biopsy.
Abdomen and chest computed tomography scan.
Regular repetition.
An initial positive phase.
An association with analgesia.
Relative audible silence.
Localization to discrete areas of muscle.
An 18-year-old male complains of weakness in the right hand. Physical examination demonstrates
an inability to flex the distal phalanx of the right thumb and normal right upper extremity sensation.
Using needle electromyography, which muscle should be tested to confirm the diagnosis?
A.
B.
C.
D.
E.
The pronator teres.
The brachioradialis.
The flexor carpi radialis.
The pronator quadratus.
The abductor pollicis longus.
2009 AANEM Electrodiagnostic Self-Assessment Examination
33
111 . In the Figure above, a response is elicited in tracings A5 through A8, that gets progressively larger.
This earliest response represents:
A.
B.
C.
D.
E.
112.
Which of the following statements about botulinum toxin is true?
A.
B.
C.
D.
E.
113.
It inhibits presynaptic calcium uptake.
It blocks postsynaptic acetylcholine receptors.
It facilitates postsynaptic magnesium uptake.
It inhibits presynaptic acetylcholine release.
It inhibits postsynaptic muscle depolarization.
Electromyographic silence is characteristic of all the following conditions EXCEPT:
A.
B.
C.
D.
E.
34
The M response.
An A wave.
An H reflex.
An F wave.
Shock (stimulus) artifact.
A muscle contracture.
An attack of hypokalemic periodic paralysis.
Myoedema due to hyperthyroid myopathy.
A cramp in phosphofructokinase deficiency.
A cramp in Isaac disease.
2009 AANEM Electrodiagnostic Self-Assessment Examination
114.
Wallerian degeneration causes:
A.
B.
C.
D.
E.
115.
The earliest common electrophysiologic abnormality encountered in an acute inflammatory
demyelinating polyneuropathy is:
A.
B.
C.
D.
E.
116.
Measure the temperature and report it, then proceed with testing.
Measure the temperature and use known correction factors.
Wait 60-90 seconds between stimulations to minimize sympathethic activation.
Compare the response to another sensory nerve in the same hand.
Warm the hand to be consistent with established normative data.
Which of the following abnormal spontaneous activities is generated by a single muscle fiber?
A.
B.
C.
D.
E.
118.
Delayed or absent F waves and H reflexes.
Small motor response amplitude.
Slowed sensory conduction velocity.
Delayed motor terminal latency.
Conduction block and temporal dispersion.
You touch your patient’s hand during an ulnar sensory nerve conduction study and note that it
feels cold. To improve the accuracy of your study, you should:
A.
B.
C.
D.
E.
117.
No change in the nerve cell body.
Rapid degeneration of the axon and myelin sheath.
Immediate loss of the distal sensory response.
Loss of Schwann cells.
Retention of the myelin sheath, but axonal degeneration.
Fasciculation potential.
Myokymic discharge.
Neuromyotonic discharge.
Myotonic discharge.
Cramp discharge.
Which of the following conditions is most likely to have X-linked inheritance?
A.
B.
C.
D.
E.
Dysferlin-associated myopathy.
Facioscapulohumeral dystrophy.
Bulbospinal muscular atrophy (Kennedy syndrome).
Charcot-Marie-Tooth disease types 1A and 1B.
Proximal myotonic myopathy.
2009 AANEM Electrodiagnostic Self-Assessment Examination
35
119.
Why is the median antidromic sensory nerve action potential (SNAP) amplitude larger than the
orthodromic amplitude?
A.
B.
C.
D.
E.
120.
When comparing the motor unit action potentials (MUAPs) detected by a monopolar needle with
those detected by a concentric needle, which of the following is most likely to be true?
A.
B.
C.
D.
E.
121.
Miniature end-plate potentials.
Ephaptic transmission among muscle fibers.
Motor unit action potentials.
Nerve twig action potentials.
Single muscle fiber action potential.
Increasing the sensitivity when recording a compound muscle action potential (CMAP) will most
likely:
A.
B.
C.
D.
E.
36
MUAP amplitudes are smaller.
An increased number of polyphasic MUAPs are seen.
A decreased number of polyphasic MUAPs are seen.
MUAP duration is longer.
MUAP duration is smaller.
End-plate noise physiologically represents:
A.
B.
C.
D.
E.
122.
There is less dispersion of the SNAP.
Stimulation is closer to the nerve itself.
Recording is closer to the nerve itself.
It is easier to achieve supramaximal stimulation.
Temperature is warmer at the wrist than the fingers.
Increase the visually measured amplitude of the CMAP.
Decrease the visually measured amplitude of the CMAP.
Increase the visually measured distal latency of the CMAP.
Decrease the visually measured distal latency of the CMAP.
Decrease the visually measured peak latency of the CMAP.
2009 AANEM Electrodiagnostic Self-Assessment Examination
123.
A 25-year-old laborer with a 1-month history of low back pain has a needle electromyography
(EMG) for possible radiculopathy. Positive waves and fibrillations are seen in the extensor hallucis
longus, gluteus medius, and mid-lumbar paraspinal muscles.
The electrodiagnostic findings most consistent with the patient’s 1-month history of radicular
symptoms would be:
A.
B.
C.
D.
E.
124.
Complex repetitive discharges in involved muscles.
Reduced recruitment in involved muscles.
Large amplitude polyphasic motor unit action potentials in involved muscles.
Absent superficial peroneal sensory responses.
Myokymic discharges in involved muscles.
A radial nerve conduction study is performed in a patient with a complete wrist drop on the day he
sustained a humeral fracture. Stimulation of the radial nerve at the elbow and recording over the
extensor indicis proprius (EIP) muscle reveals a normal compound muscle action potential
amplitude and distal latency. In this case, which of the following statements about radial nerve
injury is true?
A. A complete radial nerve injury at the spiral groove is excluded.
B. An excellent prognosis for recovery of strength is supported.
C. EIP needle electromyography studies would show fibrillation potentials if axonotmesis is
present.
D. Stimulation at the axilla would elicit no response if neurotmesis were present.
E. No useful information can be obtained at this stage after injury.
125.
If you inadvertently reverse the stimulating electrodes in an orthodromic sensory nerve conduction
study, the most notable results will be:
A.
B.
C.
D.
E.
126.
No change from what was expected.
Conduction velocity is slower than expected.
Amplitude is higher than expected.
Amplitude is lower than expected.
Latency is decreased from expected.
Which test of autonomic function best assesses both parasympathetic cholinergic afferent and
efferent function?
A.
B.
C.
D.
E.
Sympathetic skin responses.
Cold pressor test.
Valsalva maneuver blood pressure response.
Heart rate response to deep breathing.
Quantitative sudomotor axon reflex test.
2009 AANEM Electrodiagnostic Self-Assessment Examination
37
127.
Myokymic discharges are most characteristic of which of the following disorders?
A.
B.
C.
D.
E.
128.
During repetitive stimulation of the nerve at 2 to 3 times per second (Hz), the amount of
acetylcholine released into the neuromuscular junction following each depolarization of the nerve
ending:
A.
B.
C.
D.
E.
129.
Increases gradually with succeeding stimuli.
Decreases slightly as immediate stores are depleted.
Remains constant throughout stimulation.
Varies in relation to the amount of local acetylcholinesterase.
Varies in relation to the amplitude of the resulting motor response.
A 25-year-old professional cyclist developed gradual onset of weakness in the left hand, which
became worse after her last race. She must flex her thumb in order to hold onto the paper. The
picture above and description of symptoms are most consistent with the diagnosis of:
A.
B.
C.
D.
E.
38
Radiation-induced brachial plexopathy.
Neurogenic thoracic outlet syndrome.
Amyotrophic lateral sclerosis.
Leprosy.
Duchenne muscular dystrophy.
Ulnar neuropathy.
Median neuropathy.
Anterior interosseous neuropathy.
Radial nerve palsy.
Martin Gruber anastomosis and carpal tunnel syndrome.
2009 AANEM Electrodiagnostic Self-Assessment Examination
130.
While performing a left orbicularis oris needle electromyography (EMG) in a patient with a 6-month
history of chronic left facial weakness you encounter unexpected irregular brief bursts of motor
units during your search for spontaneous activity in this muscle. On closer inspection, the brief
bursts of activity may correlate with eye blinking. The best next step to document the observed
relationship would involve:
A.
B.
C.
D.
E.
131.
A low-frequency filter setting of 0.5 Hz is most appropriate when performing:
A.
B.
C.
D.
E.
132.
Upper limb F-wave studies.
Blink reflex studies.
Repetitive nerve stimulation studies.
Sural sensory nerve studies.
Sympathetic skin response studies.
During nerve conduction studies, failure to place the recording electrode (G1) over the motor point
most often results in a waveform that is:
A.
B.
C.
D.
E.
133.
Increasing the low frequency filter.
Stimulating the facial nerve at the stylomastoid foramen.
Setting the trigger to record the bursts.
Using a second EMG channel to simultaneously record from the orbicularis oculi.
Rastering sequential signals of the EMG bursts.
Monophasic, initial positive deflection.
Triphasic, initial positive deflection.
Diphasic, initial negative deflection.
Unobtainable.
Increased in amplitude.
A triphasic, high-amplitude motor unit action potential in the anterior tibialis firing in isolation at a
rate of 25 Hz would most likely be seen in a patient with:
A.
B.
C.
D.
E.
Myotonic muscular dystrophy.
Myasthenia gravis.
Polymyositis.
Prior poliomyelitis.
Complete nerve laceration.
2009 AANEM Electrodiagnostic Self-Assessment Examination
39
134.
When recording a sensory nerve action potential, the optimal distance between disc electrodes to
reduce waveform distortion from simultaneous recording of the action potential by the G 1 and G 2
electrodes is:
A.
B.
C.
D.
E.
135.
Pudendal neuropathy is associated with all of the following EXCEPT:
A.
B.
C.
D.
E.
136.
Vaginal delivery.
Skate boarding.
Bicycle riding.
Blunt pelvic trauma.
Penetrating injuries.
A decrease in limb temperature can result in all of the following EXCEPT:
A.
B.
C.
D.
E.
137.
2 cm.
2.5 cm.
3 cm.
3.5 cm.
4 cm.
Slowing of sodium channel opening.
Slowing of nerve depolarization.
Reduced nerve conduction velocity.
Slowing of sodium channel closing.
Reduced nerve action potential amplitude.
You have hired a new technician to perform nerve conduction studies (NCSs) under your
supervision. A patient is referred for evaluation of suspected bilateral median neuropathies versus
C6 radiculopathies. The patient’s hands are ice cold, 26ºC. The technician’s data show that the
bilateral median and ulnar distal latencies are mildly prolonged, and amplitudes are low normal.
Temperature correction formulas normalize the ulnar latencies, and make median latencies
borderline. What should you do?
A. Proceed to needle electromyography (EMG) and report probable mild bilateral carpal tunnel
syndrome.
B. Proceed to needle EMG and report the median NCS as borderline.
C. Perform needle EMG then repeat the NCSs at additional charge.
D. Repeat the nerve conduction studies at no extra charge after limb warming.
E. If needle EMG findings support C6 radiculopathy, conclude that the NCSs are normal.
40
2009 AANEM Electrodiagnostic Self-Assessment Examination
138.
Thigh pain followed by weakness and atrophy of the quadriceps in elderly men is most commonly
seen as a complication of which one of the following?
A.
B.
C.
D.
E.
139.
In studying a healthy nerve, if you change the temperature from 26°C to 34°C, the nerve
conduction velocity will change from 36 m/s to a value between the following ranges:
A.
B.
C.
D.
E.
140.
38 to 42 m/s.
42 to 46 m/s.
48 to 56 m/s.
58 to 62 m/s.
30 to 34 m/s.
The H reflex:
A.
B.
C.
D.
E.
141.
Alcoholism.
Uremia.
Hemophilia.
Myeloma.
Diabetes.
Is best elicited with supramaximal stimuli.
May be elicited by stimulation of many different nerves in the normal adult.
Is best elicited with stimuli of short (50 to 100 µs) duration.
Varies significantly in latency and configuration from stimulus to stimulus.
Involves stimuli that activate group insertional activity afferent fibers.
A 38-year-old male with slowly progressive leg weakness and bilateral foot drop over 5 to 6 years
is referred to the electromyography (EMG) laboratory for presumptive diagnosis of peripheral
neuropathy. Another electrodiagnostic test concluded that he had abundant distal positive sharp
waves. He has distal leg muscle atrophy and bilateral steppage gait, and denies back pain or foot
numbness. Motor and sensory nerve conduction studies are normal. Needle EMG shows many
positive sharp waves and fibrillations in the tibialis anterior and gastrocnemius muscles, and not in
the lumbar paraspinal muscles. Which of the following findings is most specific for diagnosis?
A.
B.
C.
D.
E.
Complex repetitive discharges in the lumbar paraspinal muscles.
Absent H reflexes.
Abundant myotonic discharges in distal and proximal muscles.
12% decrement in the tibialis anterior muscle by repetitive nerve stimulation.
Many polyphasic motor units in the quadriceps muscle.
2009 AANEM Electrodiagnostic Self-Assessment Examination
41
142.
You are asked to perform an electrodiagnostic evaluation in a 52-year-old female patient who
experienced the sudden onset of left hand numbness 1 week ago which resolved over 30 minutes.
The numbness involved all fingers, but was more noticeable in the thumb and index finger. Your
clinical observation discovers the presence of a mild central type facial droop, weakness in the left
deltoid and triceps muscles, and hyperreflexia. The most ethical way to proceed is:
A. Conduct a nerve conduction study to determine if there is an underlying carpal tunnel
syndrome.
B. Conduct a needle examination to determine if there is a radiculopathy.
C. Send the patient for a magnetic resonance image examination of the brain.
D. Call the referring physician to explain your concern for a central nervous system etiology.
E. Tell the patient she should urgently see a neurologist.
143.
An acquired demyelinating polyneuropathy is LEAST likely to occur in association with:
A.
B.
C.
D.
E.
144.
Which finding is most characteristic of patients with severe critical illness myopathy (acute
quadriplegic myopathy)?
A.
B.
C.
D.
E.
145.
Low amplitude compound motor action potentials.
Preserved response from direct muscle stimulation.
Absent sensory nerve action potentials.
Discrete recruitment with rapid firing rates.
Abundant fibrillation potentials in virtually all cases.
In examining a patient with a lesion of the brachial plexus, the deltoid and triceps (brachii) are
found to be weak, but the biceps (brachii) and pronator teres are found to be normal. The lesion
would be localized to the:
A.
B.
C.
D.
E.
42
Human immunodeficiency virus infection.
Anti myelin-associated glycoprotein antibody.
Diphtheria infection.
Sjögren’s syndrome.
Osteosclerotic myeloma.
Upper trunk.
Medial cord.
C6 root.
Lateral cord.
Posterior cord.
2009 AANEM Electrodiagnostic Self-Assessment Examination
146.
Which distal latency listed is closest to a normal electrical shock-elicited sympathetic skin
response recorded from the hand?
A.
B.
C.
D.
E.
147.
For the past 6 weeks, a 54-year-old school teacher has had difficulty climbing stairs, getting out of
the bathtub, and has had mild difficulty speaking. She has a subtle periorbital, anterior thorax,
periungual, and heliotrope rash. Needle electromyography will most likely demonstrate:
A.
B.
C.
D.
E.
148.
Small motor unit action potentials (MUAPs) with increased numbers firing.
Small MUAPs with decreased numbers firing.
Large MUAPs with increased numbers firing.
Large MUAPs with decreased numbers firing.
Normal MUAPs with increased numbers firing.
Sensory nerve action potential amplitude will be increased by:
A.
B.
C.
D.
E.
149.
1.5 ms.
15 ms.
150 ms.
1500 ms.
15,000 ms.
Stimulus intensity above supramaximal.
Decreasing the distance between active and reference electrodes to 2 cm.
Lowering the skin temperature to below 29°C.
Increasing the distance between stimulating and recording electrodes.
Placing the ground between the stimulating and recording electrodes.
A 56-year-old man with lower back pain radiating down to the posterior thigh to his foot, is referred
for electrodiagnostic examination. Motor and sensory nerve conduction studies in the lower
extremities are normal. Needle electromyography (EMG) of the right quadriceps, tibialis anterior,
and gastrocnemius muscles shows normal results. After needle EMG of these muscles, the patient
states that the test is too uncomfortable and he wishes to terminate the examination.
The patient calls several hours after the examination complaining of pain in the area of the muscles
examined with needle EMG. The most appropriate course of action is to:
A.
B.
C.
D.
E.
Prescribe anti-inflammatory medication.
Obtain a computerized axial tomography scan evaluation.
Prescribe narcotic analgesic medication.
Talk to the patient personally and offer to re-examine him.
Tell your office staff to have him contact his referring physician.
2009 AANEM Electrodiagnostic Self-Assessment Examination
43
150.
When performing electrodiagnostic studies on a cold limb (less than 28ºC), it would be most
difficult to correct for which one of the following to avoid a diagnostic error?
A.
B.
C.
D.
E.
Sensory distal latencies in evaluating a median entrapment at the wrist.
Conduction velocity in evaluating an axonal neuropathy.
Conduction velocity in evaluating a demyelinating neuropathy.
Conduction velocity in evaluating somatosensory evoked potentials.
Repetitive nerve stimulation in evaluating a defect in neuromuscular transmission.
Physical Medicine and Rehabilitation – please answer the questions that follow
Neurology – please answer the questions starting with 156
151.
Do you plan on taking the American Board of Electrodiagnostic Medicine (ABEM) certification
examination?
A. Yes. (proceed to question 152)
B. No. (proceed to question 153)
C. Undecided.
152.
If you answered Yes to question 151, please choose the answer that best explains your reasons
for taking the examination:
A.
B.
C.
D.
E.
It is required by my employer.
It provides credibility and respect.
It validates competence in electrodiagnostic medicine.
It improves the marketability of my practice.
None of the above.
153. If you answered No to question 151, please choose the answer that best explains your reasons for
not taking the examination:
A.
B.
C.
D.
E.
154.
It is not relevant to my future practice.
It takes too much time to prepare for the examination.
It costs too much to take the examination.
I plan to seek certification in another area.
None of the above.
Do you plan on seeking certification in another subspecialty area?
A. Yes.
B. No.
C. Undecided.
44
2009 AANEM Electrodiagnostic Self-Assessment Examination
If you answered Yes to question 154 please answer the following.
155.
I plan to seek certification in:
A.
B.
C.
D.
E.
F.
Spinal Cord Injury.
Sports Medicine.
Neuromuscular Medicine.
Pain Medicine.
Pediatric Rehabilitation.
Other subspecialty.
Please proceed to Question 161.
Neurology – please answer the questions that follow
156.
Do you plan on taking the American Board of Electrodiagnostic Medicine (ABEM) certification
examination?
A. Yes. (proceed to question 157)
B. No. (proceed to question 158)
C. Undecided.
157.
If you answered Yes to question 156, please choose the answer that best explains your reasons
for taking the examination:
A.
B.
C.
D.
E.
It is required by my employer.
It provides credibility and respect.
It validates competence in electrodiagnostic medicine.
It improves the marketability of my practice
None of the above.
158. If you answered No to question 156, please choose the answer that best explains your reasons for
not taking the examination:
A.
B.
C.
D.
E.
It is not relevant to my future practice.
It takes too much time to prepare for the examination.
It costs too much to take the examination.
I plan to seek certification in another area.
None of the above.
2009 AANEM Electrodiagnostic Self-Assessment Examination
45
159.
Do you plan on seeking certification in another subspecialty area?
A. Yes.
B. No.
C. Undecided.
If you answered Yes to question 159 please answer the following.
160.
I plan to seek certification in:
A.
B.
C.
D.
E.
Clinical Neurophysiology.
Neuromuscular Medicine.
Pain Medicine.
Sleep Medicine.
Other subspecialty.
As a young physician, the AANEM would like to understand what is important to you and how we
can better meet your needs. Please take a moment to answer the following questions:
Please rate how important the following 8 items are to you in determining whether or not you join the
AANEM. Use the following scale:
A.
B.
C.
D.
Very important.
Somewhat important.
Not important.
Don’t know.
161. Receiving discounts on AANEM educational materials (i.e., educational CD-ROMs, course books,
monographs, etc).
162. The value generated by being recognized as a member of the AANEM.
163. The opportunity to meet and network at the annual meeting.
164. The opportunity to meet and network through online communities.
165. Access to free online CME offerings such as Case Studies, Podcasts, and Journal Invited Review
articles.
166. Receiving up-to-date news on advocacy issues affecting the practice of neuromuscular and
electrodiagnostic medicine.
167. Having the AANEM be my voice with regard to coding, billing, and patient care issues.
168. Receiving a discount on the AANEM annual meeting registration.
46
2009 AANEM Electrodiagnostic Self-Assessment Examination
Please rate how familiar you are with the following AANEM products and services. Use the following
scale:
A.
B.
C.
D.
169.
170.
171.
172.
173.
174.
175.
176.
177.
Very familiar.
Familiar.
Not very familiar.
Not at all familiar.
AANEM CareerCenter – resumé posting and employment search.
Nerve & Muscle Junction – podcasts interviewing authors of recent research
Online Case Studies – FREE CME source for members
Online Invited Reviews – 3-4 yearly Muscle & Nerve articles containing information of importance
to neuromuscular and EDX physicians.
AANEM publications and resources (i.e., educational CD-ROMs, course books, monographs, etc).
AANEM Annual Meeting.
Online Presentation Slides – presentations to assist in educating referral sources about EDX
medicine.
Marketing Series – articles and worksheet to help develop a marketing plan for your
neuromuscular or EDX practice.
Do you plan on joining the AANEM?
A. Yes.
B. No.
C. Undecided.
178.
If you answered Yes to question 177, please choose the answer that best explains your reason for
joining the AANEM:
A.
B.
C.
D.
E.
179.
It enhances my credibility as an electrodiagnostic medicine or neuromuscular physician.
It provides educational opportunities at a discount or often for free.
It supports the advocacy efforts of the AANEM.
It improves the marketability of my practice.
Other reasons
If you answered No to question 177, please choose the answer that best explains your reason for not
joining the AANEM:
A.
B.
C.
D.
E.
I do not plan to practice electrodiagnostic medicine.
I do not plan to practice neuromuscular medicine.
I see no value to joining AANEM.
It costs too much to join.
Other reasons
Thank you for taking the time to answer these important questions. We value your input.
2009 AANEM Electrodiagnostic Self-Assessment Examination
47
2009 EDX SAE ANSWERS
Question 51: B
SUMMARY:
Low ulnar sensory and median motor amplitudes as well as fibrillation potentials in the abductor pollicis
brevis are findings that are present in both lower trunk and medial cord brachial plexus lesions. However,
fibrillation potentials in the extensor indicis proprius are only present in a lower trunk lesion because its
nerve fibers exit through the posterior division to join the posterior cord and continue to the radial nerve.
REFERENCE:
Preston DC, Shapiro BE. Electromyography and neuromuscular disorders, 2nd edition. Philadelphia:
Elsevier; 2005. p 480-481,490-491.
Question 52: A
SUMMARY:
The conduction velocity of an axon increases as its electrical resistance decreases. Axon diameter is
inversely related to the internal electrical resistance of the fiber. Therefore, small diameter axons have
higher axoplasmic resistance and slower conduction velocity. Larger axons also tend to be more highly
myelinated. Blood flow is unrelated to conduction velocity.
REFERENCE:
Aminoff MJ. Electrodiagnosis in clinical neurology, 4th edition. New York: Churchill Livingstone; 1999. p
15-21.
Question 53: D
SUMMARY:
The A wave is a late response that may follow a motor nerve conduction study which is generated by an
antidromic potential traveling up to a branching point and then traveling orthodromically down the axon
branch to the muscle. It is observed with submaximal stimulation and typically occurs between the M
wave and the F wave. The A wave is constant in amplitude and latency. More proximal stimulation results
in a shorter latency, indicating an initially antidromic passage of the impulse.
REFERENCE:
Kimura J. Electrodiagnosis in diseases of nerve and muscle: principles and practice, 3rd edition. New
York: Oxford University Press; 2001. p 443-445.
48
2009 AANEM Electrodiagnostic Self-Assessment Examination
Question 54: B
SUMMARY:
In lower trunk brachial plexopathies the median sensory fibers are spared because they travel through the
upper trunk, thus the median sensory nerve conduction studies would be normal. The median motor fibers
to the abductor pollicis brevis travel through the lower trunk, thus the motor nerve conduction studies are
normal in upper trunk lesions. Nerve fibers contributing to the median nerve do not traverse the posterior
cord. The anterior interosseous nerve branches distal to the pronator teres. The radial nerve can be
compressed at the Arcade of Frohse.
REFERENCE:
Dumitru D, Amato AA, Zwarts MJ. Electrodiagnostic medicine, 2nd edition. Philadelphia: Hanley & Belfus;
2002. p 790-794.
Question 55: A
SUMMARY:
Cool limb temperature results in slowing of distal latencies and conduction velocities, an increase in the
amplitude of the sensory nerve action potentials more than the compound muscle action potentials, and
an increase in the duration, amplitude, and phases of motor unit action potentials. Also, cool limb results
in improving neuromuscular transmission which may lead to an erroneously normal repetitive stimulation.
REFERENCES:
Denys EH. AAEM Minimonograph #14: The influence of temperature in clinical neurophysiology. Muscle
Nerve 1991;14:795–811.
Dumitru D, Amato AA, Zwarts M. Electrodiagnostic medicine, 2nd edition. Philadelphia: Hanley & Belfus;
2002. p 188-189.
Question 56: B
SUMMARY:
Slow repetitive stimulation studies in myasthenia gravis typically show the greatest decrement between
the first and second response; however, the fourth or fifth potentials are the typical comparison waveform.
The magnitude of the decremental response in myasthenia gravis is reduced in a cool limb.
Anticholinesterase drugs may influence the results of repetitive stimulation studies and should be
discontinued in preparation for these tests. Facilitation is not typically seen in slow repetitive stimulation
studies in myasthenia gravis and a decrement is generally considered abnormal when it reaches 10% in a
distal muscle and may be higher in a proximal muscle.
REFERENCE:
Kimura J. Electrodiagnosis in diseases of nerve and muscle: principles and practice, 3rd edition. New
York: Oxford University Press; 2001. p 263-265.
Question 57: D
SUMMARY:
Motor conduction velocities are approximately 50% of adult values at birth, but gradually increase due to
myelination of the nerve fibers and reach adult values by 3 to 5 years of age.
REFERENCE:
Kimura J. Electrodiagnosis in diseases of nerve and muscle: principles and practice, 3rd edition. New
York: Oxford University Press; 2001. p 11-112.
2009 AANEM Electrodiagnostic Self-Assessment Examination
49
Question 58: B
SUMMARY:
The earliest abnormality detected in a radiculopathy is decreased motor unit recruitment, which can be
seen from the onset of symptoms. Fibrillations do not typically appear until later and start in more proximal
muscles, such as the paraspinal muscles. However, this activity occurs no earlier than 10 to 14 days and
often not before 3 weeks. Increase in motor unit action potential amplitude and duration are seen many
weeks after onset as a result of collateral sprouting and reinnervation. Increased or early motor unit
recruitment is found in myopathic conditions and is not an expected finding in radiculopathy.
REFERENCE:
Dumitru D, Amato AA, Zwarts MJ. Electrodiagnostic medicine, 2nd edition. Philadelphia: Hanley & Belfus;
2002. p 732, 738-745.
Question 59: A
SUMMARY:
Myokymic discharges favor the diagnosis of radiation-induced plexopathy. Fibrillation potentials may
result from axonal injury due to either radiation or tumor infiltration. Axonal loss in the lower brachial
plexus would reduce or eliminate the sensory nerve action potential from the ulnar and medial
antebrachial cutaneous nerves. Painful lower trunk plexopathy with Horner’s syndrome favors tumor
infiltration, whereas painless upper trunk lesions with lymphedema suggests radiation injury.
REFERENCE:
Kimura J. Electrodiagnosis in diseases of nerve and muscle, 3rd edition. New York: Oxford University
Press; 2001. p 633-634.
Question 60: C
SUMMARY:
A constant current stimulator is the preferred device, for nerve conduction studies as it delivers a
consistent stimulus current that is independent of impedance. The voltage across the stimulating
electrode adjusts dynamically to maintain a constant current stimulus despite potentially changing skin
impedance. A constant voltage stimulator has a fixed voltage output. As such changes in skin impedance
alter the stimulus current level. This can result in inconsistent stimulus intensity and sub maximal
stimulation.
REFERENCE:
Kimura J. Electrodiagnosis in diseases of nerve and muscle: principles and practice, 3rd edition. New
York: Oxford University Press; 2001. p 53.
Question 61: E
SUMMARY:
Perifascicular atrophy is highly specific for dermatomyositis and is frequently evident in more chronic
disease stages. The process is thought to be a reflection of hypoperfusion that occurs from local
endomysial microvasculature changes, leading to muscle fiber destruction and inflammation.
Perifascicular atrophy is characterized by 2 to 10 layers of atrophic fibers at the periphery of the fascicles.
This pattern is not evident in other chronic muscular dystrophies or inflammatory myopathies.
REFERENCES:
Griggs RC, Mendell JR, Miller RG. Evaluation and treatment of myopathies. Philadelphia: FA Davis; 1995.
p 159–160.
Katirji B, Kaminski HJ, Preston DC, Ruff RL, Shapiro BE. Neuromuscular disorders in clinical practice.
Boston: Butterworth Heinemann; 2002. p 1175-1176.
50
2009 AANEM Electrodiagnostic Self-Assessment Examination
Question 62: A
SUMMARY:
After giving off the dorsal cutaneous branch 6-8 cm proximal to the ulnar styloid, the ulnar nerve enters
the distal ulnar tunnel, termed Guyon’s canal, where it divides into superficial and deep palmar branches.
The proximal Guyon’s canal entrapment causes ulnar palmar sensory loss and weakness of all ulnar
intrinsic hand muscles. Distal Guyon’s canal entrapment can produce weakness of all ulnar intrinsic hand
muscles without sensory loss. Deep palmar branch entrapment at the pisohamate hiatus causes
weakness of ulnar intrinsic hand muscles with sparing of the hypothenar muscles,but no sensory loss.
Entrapment at the mid palm causes weakness of the adductor pollicis, the first, second and possibly the
third interossei and spares the fourth interosseous, hypothenar muscles, sensory function. The ligament
of Struthers causes a proximal median neuropathy; the arcade of Frohse affects the radial nerve; the
spiral groove is a site of radial nerve compression, and the transcarpal ligament is part of the carpal tunnel
that can compress the median nerve in the wrist.
REFERENCE:
Katirji B. Electromyography in clinical practice: a case study approach, 2nd edition. Philadelphia: Elsevier;
2007. p 86.
Question 63: C
SUMMARY:
Following axon loss plexopathy (as well as any other mixed peripheral nerve lesion), Wallerian
degeneration progresses following the distal compound muscle action potential then falls precipitously to
reach its nadir by 5–6 days post injury. In contrast to the motor studies, the distal sensory nerve remains
excitable for a slightly longer period. The distal sensory nerve action potential remains normal for 5–6
days and then decreases rapidly to reach its nadir in 10–11 days. Fibrillation potentials appear later at
around 10-14 days and become full after 3 weeks. Polyphasic motor unit action potentials become
apparent a month after the injury.
REFERENCE:
Katirji B. Electromyography in clinical practice: a case study approach, 2nd edition. Philadelphia: Elsevier;
2007. p 51-52.
Question 64: D
SUMMARY:
As the median nerve descends in the upper arm, it runs medial to the humerus and anterior to the medial
epicondyle. An uncommon tendinous band known as the ligament of Struthers stretches between the spur
and medial humeral epicondyle and can compress the median nerve prior to the first motor branch. In the
antecubital fossa the median nerve travels adjacent to the brachial artery. As it enters the forearm it runs
beneath the lacertus fibrosis, a thick fibrous band that runs from the medial aspect of the biceps tendon to
the proximal forearm flexor musculature. The median nerve can also be entrapped in the substance of
pronator teres muscle, which will affect all median muscles except for the pronator teres itself; the next
innervated and typically affected muscle is the flexor carpi radialis. More distally the nerve can be
entrapped beneath the sublimis bridge of flexor digitorum sublimis muscle. The brachialis muscle is not
associated with median nerve entrapment.
REFERENCE:
Preston DC, Shapiro BE. Electromyography and neuromuscular disorders, 2nd edition. Philadelphia:
Elsevier; 2005. p 256.
2009 AANEM Electrodiagnostic Self-Assessment Examination
51
Question 65: A
SUMMARY:
This patient has findings that suggest a local process other than carpal tunnel syndrome. In order to
completely evaluate the problem, a more complete ipsilateral study and possibly selective comparisons to
the contralateral side are needed; the electrodiagnostic (EDX) medicine physician has reasonable
discretion about what studies are appropriate to assess specific clinical problems that may change
depending on the EDX data gathered. It is not necessary to gain supplementary permission from the
referring physician to complete the assessment. However, it is not reasonable to perform an excessive
number of tests on asymptomatic regions without a clear indication. Contacting the referring physician in
order to discuss a study is reasonable if the initial study indication is unclear or if your clinical impression
varies greatly with theirs. Some examples include a patient with a well known, but independent
neuromuscular condition that is not at issue, or a patient that you believe has a serious generalized
disorder that would not be adequately assessed by the study requested.
In addition it is inappropriate for you to criticize the prior decisions made in this setting even if you are
confident of your opinions because you, as a consultant, are likely also have inadequate clinical
information and records. It is also inappropriate to discuss recommended treatments directly with the
patient. If the patient is additionally referred to you for a clinical evaluation and adequate medical records
are supplied, the consultant should still constrain critical judgments of other physicians to questions within
their area of specialty or expertise. Although verbally communicating treatment opinions and concerns
directly to the referring physician are reasonable, the findings in this case do not suggest severe median
neuropathy at the wrist.
REFERENCES:
Bolton, Charles F. Neuromuscular manifestations of critical illness. Muscle and Nerve. 2005;32:140-163.
Robinson, LR. Trauma to Peripheral Nerves. Suppl Clin Neurophysiol. 2004;57:173-186.
Williams, FH, editor. Self-Directed Physiatric Education Program (SDPEP), Study Guide and SelfAssessment Exam-Practitioners (SAE-P), 2005, the sixth edition. Arch Phys Med Rehab. 2005;86:
S1-85.
Question 66: B
SUMMARY:
This question concerns the accessory deep peroneal nerve. The only abnormality in this study is the left
peroneal nerve distal compound muscle action potential (CMAP) amplitude, which is smaller than the
proximal amplitude and which falls below the lower limit of normal. Common reasons for decreased distal
amplitude is either failure to obtain supramaximal stimulation, proximal spread of stimulation to the tibial
nerve behind the knee, or anomalous innervation of the extensor digitorum brevis (EDB) by the superficial
peroneal nerve. Obtaining a CMAP response by stimulating behind the lateral malleolus and activating the
anomalously innervated part of the EDB will help explain the distal drop in the CMAP amplitude. This
anomaly affects approximately 20% of the population.
REFERENCE:
Dumitru D. Electrodiagnostic medicine. Philadelphia: Hanley & Belfus; 1995. p 434.
52
2009 AANEM Electrodiagnostic Self-Assessment Examination
Question 67: C
SUMMARY:
The lateral antebrachial cutaneous (LAC) nerve is the terminal sensory branch of the musculocutaneous
nerve and supplies sensation to the lateral forearm. It originates from the lateral cord of the brachial
plexus and innervates the coracobrachialis, biceps brachii, and brachialis. It does not communicate with
the median nerve. Distal musculocutaneous neuropathies at the elbow only affect the LAC nerve,
therefore the needle electromyography examination is normal.
REFERENCE:
Dumitru D, Amato AA, Zwarts MJ. Electrodiagnostic medicine, 2nd edition. Philadelphia: Hanley & Belfus;
2002. p 783, 809-810.
Question 68: E
SUMMARY:
A phase is the portion of a motor unit action potential (MUAP) that departs from, and returns to, the trace
baseline. A turn is a MUAP directional change without crossing the baseline. A MUAP is called polyphasic
when it has more then four phases, or serrated when it has more than five turns. A MUAP is considered
complex when it is either polyphasic, serrated, or both. Increased complexity indicates desynchronization
of single muscle fiber discharges.
REFERENCE:
Dumitru D, Amato AA, Zwarts MJ. Electrodiagnostic medicine, 2nd edition. Philadelphia:Hanley & Belfus;
2002. p 272, 297-298.
Question 69: C
SUMMARY:
With median to ulnar cross over (Martin-Gruber anastomosis), fibers that normally ulnar muscles travel
with the median nerve then cross to the ulnar nerve in the forearm; subtypes include (1) fibers directed to
the hypothenar, (2) first dorsal interosseous, and (3) ulnar thenar muscles. In these situations, the ulnar
motor nerve conduction study recording at the first dorsal interosseous (FDI) or abductor digiti minimi
(ADM), manifests a drop in the ulnar compound motor action potential (CMAP) amplitude between distal
and proximal stimulation sites (false conduction block). With distal stimulation at the wrist, the CMAP
reflects all ulnar motor fibers, while proximal stimulations (usually both below and above elbow) activate
only the lesser number of uncrossed fibers. This anomaly can be confirmed by median nerve stimulation
at the wrist elbow while still recording over the ADM or FDI that evokes a significant CMAP proximally but
not distally. There is no median conduction block and velocities in the forearms are not slowed. In these
examples the median amplitude increases, not decreases with proximal stimulation.
REFERENCE:
Katirji B. Electromyography in clinical practice: a case study approach, 2nd edition. Philadelphia: Elsevier;
2007. p 19-20.
2009 AANEM Electrodiagnostic Self-Assessment Examination
53
Question 70: A
SUMMARY:
The N13 response is a near-field potential obtained after stimulation of the median or ulnar nerve. It
represents a stationary, nonpropagated signal in the cervical cord recorded form the dorsal neck. The
other responses are generated in the brain stem and represent propagated waves that are recorded from
a distance (scalp) and thus are far-field potentials.
REFERENCE:
American Association of Electrodiagnostic Medicine. Guidelines for somatosensory evoked potentials.
Muscle Nerve 1999;22:S123-S138.
Question 71: C
SUMMARY:
Complex repetitive discharges typically start and stop abruptly. Fibrillation potentials and positive waves
typically fire in a regular pattern. Myotonic discharges are triggered by needle movement with a waxing
and waning pattern. Myokymic discharges are regular and persistent discharges.
REFERENCE:
Preston DC, Shapiro BE. Electromyography and neuromuscular disorders, 2nd edition. Philadelphia:
Elsevier; 2005. p 207.
Question 72: B
SUMMARY:
After branching off from the upper trunk of the brachial plexus, the suprascapular nerve traverses the
suprascapular notch to enter the supraspinous fossa, innervating the supraspinatus muscle. It then
passes through the spinoglenoid notch to enter the infraspinous fossa and innervate the infraspinatus
muscle. Entrapment at this level would result in isolated atrophy and weakness of the infraspinatus.
REFERENCE:
Dumitru D, Amato AA, Zwarts MJ. Electrodiagnostic medicine, 2nd edition. Philadelphia: Hanley & Belfus;
2002. p 805-806.
Question 73: C
SUMMARY:
With the sweep speed set at 10 ms/division on a 10 division screen, the total sweep duration is 100 ms.
Assuming a negligible time period for the sweep to return and sweep again, there will be 10 sweeps per
second. If a single voluntary motor unit appears on each sweep and remains stationary, one can multiply
by a factor of 10 for a firing frequency of 10 Hz. If the same motor unit appears twice on each sweep
remaining stationary, the frequency is 20 Hz. If the potentials appear to be moving slowly to the left with
each sweep, they are firing faster than 20 Hz. If they appear to move slowly to the right with each sweep,
they are firing slower than 20 Hz.
REFERENCE:
Dumitru D, Amato AA, Zwarts MJ. Electrodiagnostic medicine, 2nd edition. Philadelphia: Hanley & Belfus;
2002. p 272-274.
54
2009 AANEM Electrodiagnostic Self-Assessment Examination
Question 74: D
SUMMARY:
Extensor digitorum communis (C7, C8) and pronator teres (C6, C7) both receive significant innervation
from C7. Anconeus is innervated by C7 and C8, while biceps brachii receives innervation from C5 and C6
(no C7). Triceps receives significant C7 innervation (as well as C8 and, to lesser degree, C6), while
brachioradialis receives C5 and C6 innervation. Abductor pollicis longus in innervated by C7 and C8, but
pronator quadratus receives no C7 innervation (C8, T1). Flexor carpi radialis receives primarily C6 and C7
innervation (and to a lesser extent C8).
REFERENCE:
Dumitru D, Amato AA, Zwarts MJ. Electrodiagnostic medicine, 2nd edition. Philadelphia: Hanley & Belfus;
2002. p721.
Question 75: C
SUMMARY:
The most characteristic antibodies in the Lambert-Eaton myasthenic syndrome (LEMS) attack presynaptic
voltage-gated calcium channels (VGCCs) and impair the influx of calcium ions during nerve terminal
depolarization and ultimately inhibiting acetylcholine release. Using a standard radioimmunoassay, it is
possible to detect antibodies against VGCCs in significant titers in more than 90% of LEMS patients.
Inhibition or inactivation of synaptobrevin or vesicle-associated membrane protein is associated with
various serotypes of botulinum toxin.
REFERENCE:
Katirji B, Kaminski HJ, Preston DC, Ruff RL, Shapiro BE. Neuromuscular disorders in clinical practice.
Boston: Butterworth Heinemann; 2002. p 934.
Question 76: B
SUMMARY:
A dedicated power circuit and isolated ground minimizes the noise that is added to the power line from
other equipment in the vicinity. As such no other appliances share the same circuit as the
electromyography machine. This will dramatically decrease interference and noise.
REFERENCE:
Kimura, J. Electrodiagnosis in diseases of nerve and muscle: principles and practice, 3rd edition. New
York: Oxford University Press; 2001. p 51-52.
Question 77: D
SUMMARY:
The likelihood of detecting electrophysiological abnormalities in a patient with myasthenia gravis (MG) is
greater (1) when proximal muscles are evaluated, either via single fiber electromyography (SFEMG) or
when performing repetitive stimulation, (2) with greater disease severity, and (3) with a greater number of
recording sites. SFEMG reveals abnormalities in patients with MG 92%-100% of the time, repetitive
stimulation 77%-100% of the time, and anti-acetylcholine receptor antibodies 73%-80% of the time.
Decremental responses are seen with slow rates of stimulation, notably 2-3 Hz. Blocking is a
characteristic phenomenon in neuromuscular junction disease, but increased jitter is a more sensitive
abnormality, which can be present in clinically unaffected muscles. The finding, however, is not highly
specific, and may be found in a variety of other neuromuscular disorders.
REFERENCE:
Dumitru D, Amato AA, Zwarts MJ. Electrodiagnostic medicine, 2nd edition. Philadelphia: Hanley & Belfus;
2002. p 1172.
2009 AANEM Electrodiagnostic Self-Assessment Examination
55
Question 78: B
SUMMARY:
Hereditary neuropathy with liability for pressure palsy is characterized by multiple and recurrent
entrapment neuropathies. A characteristic pattern of findings on electrodiagnostic studies is reported. The
genetic basis is a deletion of the PMP-22 gene on chromosome 17p11.2. Duplication of this same region
results in the most common form of Charcot-Marie-Tooth (CMT) disease, CMT1A. Defects in the
transthyretin gene are associated with hereditary amyloidosis, which often is associated with median
neuropathy at the wrist, but not other entrapment syndromes. Methylmalonic acid levels are increased
with B12 deficiency. GM1 ganglioside antibodies are associated with various syndromes, including
multifocal motor neuropathy, characterized by focal neuropathy not at typical entrapment sites; acute
axonal neuropathy or axonal Guillain-Barré syndrome is also seen with these antibodies. This picture is
not an identifiable paraneoplastic syndrome to suspect an underlying lung cancer.
REFERENCES:
Li J, Krajewski K, Shy ME, Lewis RA. Hereditary neuropathy with liability to pressure palsy: the
electrophysiology fits the name. Neurology 2002;58:1769-1773.
Shy ME. Hereditary neuropathies. in Rowland LP, editor. Merritt’s textbook of neurology, 11th edition.
Baltimore: Williams & Wilkins; 2005. p 738-747.
Question 79: A
SUMMARY:
Duchenne muscular dystrophy is the only entity listed that reveals characteristic electromyography
features of myopathy with low-amplitude, short-duration motor unit action potentials (MUAPs). Muscle
fiber size variation contributes to the increase in MUAP’s phase characteristics. Large amplitude MUAPs
are seen in old poliomyelitis, adult-onset spinal muscular atrophy, and chronic inflammatory demyelinating
polyneuropathy. An acute radiculopathy would not show changes in MUAP amplitude.
REFERENCE:
Dumitru D, Amato AA, Zwarts MJ. Electrodiagnostic medicine, 2nd edition. Philadelphia:Hanley & Belfus;
2002. p 538, 952, 1275.
Question 80: D
SUMMARY:
The unstable baseline is indicative of movement artifact. Despite technical difficulties of movement artifact
proximal muscles must be included in repetitive stimulation studies for an adequate test of neuromuscular
junction function. Distal muscles while easier to immobilize have a lower sensitivity. Lowering the
temperature may mask a decrementing response to slow rates of repetitive stimulation by slowing sodium
channel opening and delaying closing.
REFERENCE:
Kimura, J, Electrodiagnosis in diseases of nerve and muscle: principles and practice, 3rd edition. New
York: Oxford University Press; 2001. p 753-758.
56
2009 AANEM Electrodiagnostic Self-Assessment Examination
Question 81: B
SUMMARY:
In normal conditions, every muscle fiber within a motor unit fires with each motor neuron discharge and
thus motor unit action potential (MUAP) amplitude does not vary with repeated discharges. With every
discharge, all neuromuscular junctions transmit the impulse and all muscle fibers generate an action
potential. Thus, the summated potential (MUAP) should remain stable. Moment to moment variation of
MUAP amplitude is highly suggestive of a neuromuscular junction disorder. Repetitive stimulation studies
are indicated and single fiber electromyography may be helpful if these studies are inconclusive. Cool
temperature often improves neuromuscular transmission. The instability is not produced by low frequency
filter settings and alerting the setting will affect the measured waveform, but not the stability.
REFERENCE:
Oh SJ. Electromyography: neuromuscular transmission studies, 2nd edition. Boston: Little Brown; 1989. p
135–136.
Question 82: A
SUMMARY:
The concomitant presence of myopathy and peripheral neuropathy with superimposed bilateral carpal
tunnel syndrome (CTS) in a previously healthy young woman with recent weight gain should raise
suspicion for hypothyroid state. Most patients with severe hypothyroid myopathy have myxedema and
clinical or subclinical peripheral neuropathy including CTS. The myopathy and electrophysiological
abnormalities improve with treatment of the hypothyroidism. Hypothyroidism may be found in up to 6% of
myasthenia gravis patients, but usually not associated with neuropathy or CTS. Muscle weakness is rarely
the presenting manifestation of endogenous hypercortisolism. In these patients, sensation and deep
tendon reflexes are spared.
REFERENCES:
Oh SJ. Principles of clinical electromyography: case studies. Baltimore: Williams & Wilkins; 1998. p 410.
Dumitru D, Amato AA, Zwarts MJ. Electrodiagnostic medicine. 2nd edition. Philadelphia: Hanley & Belfus;
2002. p 1394-1397.
Question 83: E
SUMMARY:
The blink reflex is analogous to the corneal reflex in clinical practice. The trigeminal nerve is the afferent
limb of the reflex arc, whereas the facial nerve, bilaterally, is the efferent limb of the arc. Stimulation of the
supraorbital sensory nerve fibers elicits an early R1 component only on the side of stimulation and a late
bilateral R2 component. The R1 component is a pontine reflex, while the R2 component is presumably
relayed through a more complex route including the pons and lateral medulla.
REFERENCE:
Kimura J. Electrodiagnosis in disease of nerve and muscle, 3rd edition. New York: Oxford University
Press; 2001. p 409–410.
2009 AANEM Electrodiagnostic Self-Assessment Examination
57
Question 84: A
SUMMARY:
After an acute radiculopathy, fibrillation potentials appear in the muscles that are most proximal to the
lesion (paraspinal muscles) followed by proximal limb muscles and then distal limb muscles.
Reinnervation changes, resulting in polyphasic motor unit action potentials (MUAPs) and large amplitude
and increased duration MUAPs appear later.
REFERENCE:
Preston DC, Shapiro BE. Electromyography and neuromuscular disorders, 2nd edition. Philadelphia:
Elsevier; 2005. p 466-467.
Question 85: A
SUMMARY:
The femoral nerve and obturator nerve are the two largest branches of the lumbar plexus and innervate
rectus femoris and gracilis, respectively.
REFERENCE:
Dumitru D, Amato AA, Zwarts MJ. Electrodiagnostic medicine, 2nd edition. Philadelphia: Hanley & Belfus;
2002. p 838-845.
Question 86: D
SUMMARY:
As reinnervation occurs, the degree of abnormal spontaneous activity (fibrillations and positive sharp
waves) begins to diminish, though will not completely disappear for possibly months. Low-amplitude
polyphasic motor unit action potentials (sometimes called nascent potentials) are early markers of
reinnervation due to the small number of muscle fibers supplied by the developing terminal sprouts which
are immaturely myelinated. Nerve conduction velocity may remain slowed for prolonged periods, even
after reinnervation is established.
REFERENCE:
Dumitru D, Amato AA, Zwarts MJ. Electrodiagnostic medicine, 2nd edition. Philadelphia: Hanley & Belfus;
2002. p 281-283.
Question 87: C
SUMMARY:
The peroneal division of the sciatic nerve is injured much more frequently than the tibial component. This
may clinically appear to be a peroneal neuropathy, but needle electromyography abnormalities involving
the short head of the biceps femoris—innervated by the peroneal division of the sciatic nerve – would
confirm that the lesion is located proximal to the formation of the common peroneal nerve. A low
amplitude or absent sensory nerve action potential localizes an axon-loss nerve injury to somewhere
distal to the dorsal root ganglion. A severe lesion involving the entire L5 nerve root or anterior horn cells
could result in an unobtainable compound muscle action potential (CMAP) from the extensor digitorum
brevis. The same would be expected for a severe lesion involving the S1 nerve root or anterior horn cells,
resulting in an absent CMAP from the abductor hallucis. A normal sural latency could be expected with an
S1 root lesion, and would also be normal with a lesion distal to the S1 dorsal root ganglion, unless
substantial axon loss—including the fastest conducting fibers—occurred. An absent H reflex can result
from a lesion involving axons at or distal to the nerve root level.
REFERENCE:
Kimura J. Electrodiagnosis in diseases of nerve and muscle, 3rd edition. New York: Oxford University
Press, Inc; 2001. p 730.
58
2009 AANEM Electrodiagnostic Self-Assessment Examination
Question 88: B
SUMMARY:
The amplitude and latency of consecutive motor evoked potentials elicited by the same stimulus varies
over time, although latency is much more stable during either relaxation or contraction. Preceding
voluntary contraction is the most important variable used to stabilize the responses (5% to 10 % maximum
voluntary contraction is sufficient to greatly reduce threshold, increase amplitude, and significantly shorten
response latency). Using a round coil centered at the vertex, induced monophasic current in the brain
directed in a posterior-to-anterior direction, optimally stimulates motor cortex compared to the opposite
direction. Repetitive excitatory postsynaptic potentials mediated by multiple I waves originating in cortex,
through temporal summation, achieves motor threshold.
REFERENCES:
Kimura J. Electrodiagnosis in diseases of nerve and muscle: principles and practice, 3rd edition. New
York: Oxford University Press; 2001. p 554-566.
Levin KH, Luders HO, Comprehensive Clinical Neurophysiology. Philadelphia: WB Saunders; 2000. p
330-333.
Question 89: C
SUMMARY:
Raising the low frequency filter to 500Hz will selectively remove the low frequency component of the
compound muscle action potential. The amplitude is primarily determined by low frequency components.
Rise time and latency are determined by high frequency components.
REFERENCE:
Kimura, J. Electrodiagnosis in diseases of nerve and muscle: principles and practice, 3rd edition. New
York: Oxford University Press; 2001. p 98-100.
Question 90: D
SUMMARY:
Conduction along the nerve distal to an axon loss lesion may be preserved for up to 7 days, but by 10 to
14 days the amplitude of the compound muscle action potential will decrease in proportion to the
completeness of the axon loss injury. Performing nerve conduction studies above and below the fibular
head prior to that time would disclose an apparent block of conduction, but one would not be able to
differentiate a non-continuity block of conduction due to an axon loss lesion from a demyelinating
conduction block (neurapraxia).
REFERENCE:
Kimura J. Electrodiagnosis in diseases of nerve and muscle: principles and practice, 3rd edition. New
York: Oxford University Press; 2001. p 9-103.
Question 91: A
SUMMARY:
Each physician has an obligation to share in providing care for the indigent; caring for the poor should be
a regular part of a physician’s practice.
REFERENCE:
American Medical Association. Code of medical ethics: current opinions with annotations. 1998-1999
edition. Chicago: American Medical Association; 1998. p 169.
2009 AANEM Electrodiagnostic Self-Assessment Examination
59
Question 92: B
SUMMARY:
This description is characteristic of myotonic dystrophy with early life cataracts, facial weakness, and
distal muscle weakness. Needle electromyography is most useful because it will likely show myotonic
discharges; however, genetic testing has supplanted Electrodiagnostic studies in many instances for this
disorder. Nerve conduction studies and nerve biopsy are usually normal or show minimal signs of
neuropathy. Muscle biopsy shows nonpathognomic changes, such as central nuclei and creatine kinase is
modestly elevated or shows mild nonspecific changes.
REFERENCE:
Katirji B, Kaminski HJ, Preston DC, Ruff RL, Shapiro BE. Neuromuscular disorders in clinical practice.
Boston: Butterworth-Heinemann; 2002. p 1078-1084.
Question 93: A
SUMMARY:
Neurogenic thoracic outlet syndrome is caused by compression of the lower trunk of the brachial plexus.
Nerve conduction abnormalities found in these patients consist of reduced of absent ulnar sensory nerve
action potentials (SNAPs) and decreased compound muscle action potentials of the abductor pollicis
brevis and ulnar intrinsic hand muscles when compared with the normal side. In middle trunk plexopathy,
median SNAPs are usually abnormal as well.
REFERENCE:
Dumitru D, Amato AA, Zwarts MJ: Electrodiagnostic medicine: 2nd edition. Philadelphia: Hanley & Belfus;
2002. p 817-820.
Question 94: D
SUMMARY:
The clinical findings likely indicate a brachial plexopathy. Needle electromyography evidence of
denervation in hand intrinsic muscles with absent median and ulnar responses on motor nerve conduction
studies indicates axonal loss in C8 - T1 fibers. F responses will be abnormal in either case. Sensory
responses will be abnormal or absent in postganglionic injury, i.e., brachial plexopathy. The ulnar nerve
has exclusive C8 - T1 derived fibers, unlike the median nerve, which receives fibers from the upper and
middle trunk (C6, C7) including sensory fibers to the index finger. A normal ulnar sensory response infers
a preganglionic lesion, such as a root avulsion or nerve root injury. In addition testing of the medial
antebrachial cutaneous nerve is also useful, especially if T1 fibers are preferentially affected.
REFERENCE:
Dumitru D, Amato AA, Zwarts MJ. Electrodiagnostic medicine, 2nd edition. Philadelphia: Hanley & Belfus;
2002. p 790-792.
Question 95: B
SUMMARY:
Motor units that are composed of type I muscle fibers have a lower threshold for recruitment compared to
type II muscle fibers. Type I motor units have slower conduction velocities, smaller diameter axons, slower
firing frequencies, and lower frequencies of miniature end-plate potentials.
REFERENCE:
Kimura J. Electrodiagnosis in diseases of nerve and muscle: principles and practice, 3rd edition. New
York: Oxford University Press; 2001, p 291-292.
60
2009 AANEM Electrodiagnostic Self-Assessment Examination
Question 96: E
SUMMARY:
The constellation of electrodiagnostic findings noted above is consistent with the Riche-Cannieu anomaly,
a motor anastomosis between the median and ulnar nerve in the hand, resulting in the so-called “all ulnar
hand”. This affects only motor fibers and is typically recognized when the median motor response is
reduced with normal needle electromyography of thenar muscles. Stimulating the ulnar nerve at the wrist
while recording from the thenar eminence provides a normal motor response, documenting an essentially
“all ulnar hand”.
REFERENCE:
Dumitru D, Amato AA, Zwarts MJ. Electrodiagnostic medicine, 2nd edition. Philadelphia: Hanley and
Belfus; 2002. p 192-193.
Question 97: B
SUMMARY:
Lambert-Eaton myasthenic syndrome (LEMS) is characterized by reduced release of acetylcholine from
the presynaptic terminals. Patients present with proximal and minimal to no ocular or bulbar weakness. It
is frequently paraneoplastic most commonly associated with small cell lung carcinoma. Forty to fifty
percent of LEMS patients ultimately develop cancer. All patients with LEMS should be screened for
cancer particularly lung cancer. Acetylcholine release is transiently increased by brief exercise
(posttetanic facilitation). Antibodies to voltage-gated calcium channels are found in nearly all patients. The
syndrome is not associated with muscle specific kinase antibodies.
REFERENCE:
Katirji B. Electromyography in clinical practice: a case study approach, 2nd edition. Philadelphia: Elsevier;
2007. p 237.
Question 98: C
SUMMARY:
The dorsal scapular nerve usually arises directly from the C5 spinal nerve root and innervates the
rhomboids.
REFERENCE:
Dumitru D, Amato AA, Zwarts MJ. Electrodiagnostic medicine, 2nd edition. Philadelphia: Hanley & Belfus;
2002. p 781-782.
Question 99: B
SUMMARY:
The H-reflex amplitude is maximum with submaximal stimulation, in contrast to both the F and M wave
responses, which require supramaximal stimulation. The H reflex arises from a monosynaptic reflex arc
and displays a stable amplitude with slow rates of stimulation. The H reflex is recorded from the tibial
nerve/gastrocnemius/soleus muscles as well as from the median nerve/flexor carpi radialis muscle.
Backfiring of motor neurons is the mechanism of the F wave.
REFERENCE:
Dumitru D, Amato AA, Zwarts MJ. Electrodiagnostic medicine, 2nd edition. Philadelphia: Hanley and
Belfus; 2002. p 244-249.
2009 AANEM Electrodiagnostic Self-Assessment Examination
61
Question 100: C
SUMMARY:
Inclusion body myositis is the most common acquired inflammatory myopathy in patients over age 50 and
demonstrates a propensity to affect quadriceps and finger flexor muscles among others. The asymmetry
is unusual compared to many other forms of hereditary and acquired myopathy. Rimmed vacuoles are the
characteristic pathological finding; vacuoles contain fibrillar material that include beta-amyloid and stains
with traditional amyloid stains such as Congo red and thioflavin S. Amyloid myopathy, which
demonstrates discrete amyloid collection not specific vacuoles, is a very rare complication of generalized
amyloidosis. Patchy dystrophin staining is typical of Becker type muscular dystrophy, which is typically
symmetric with a much earlier age of onset. Nemaline rods are found in several forms of hereditary
myopathy that can present at this age but are also typically symmetric and proximal. Intramuscular
indications of vasculitis are seen in patients with dermatomyositis, which is typically more symmetric,
more proximal, and less common in this age group. Renaut bodies are round or elliptical hyaline bodies
found in the endoneurium of peripheral nerves. They may serve as minute cushions to absorb pressure
and are preferentially increased at sites of nerve entrapment.
REFERENCE:
Preston DC, Shapiro BE. Electromyography and neuromuscular disorders, 2nd edition, Philadelphia:
Elsevier; 2005. p 582-583, 586-589.
Question 101: B
SUMMARY:
Decreasing the limb temperature will result in an increased sensory nerve action potential (SNAP)
amplitude secondary to cold-induced slowing of sodium channel opening and a delay of its activation.
Supramaximal stimulation results in activation of all of the nerve fibers being stimulated. Further increase
in the stimulation intensity beyond the supramaximal level will not change the amplitude of the response,
as all nerve fibers are already activated. Increasing the distance between the stimulating and the
recording electrode will result in decreased SNAP amplitude. Decreasing the high-frequency filter will
increase the onset latency and impact rise time.
REFERENCE:
Preston DC, Shapiro BE. Electromyography and neuromuscular disorders, 2nd edition. Philadelphia:
Elsevier; 2005. p 104-110.
Question 102: C
SUMMARY:
Miniature end-plate potentials (MEPPs) represent spontaneous depolarizations of the postsynaptic
membrane caused by sustained but random release of quanta of acetylcholine from the presynaptic
terminal. The amplitude of the MEPP remains normal in Lambert-Eaton myasthenic syndrome and in
botulism although the rate of release of the MEPPs is reduced in these disorders.
REFERENCE:
Kimura J. Electrodiagnosis in diseases of nerve and muscle: principles and practice, 3rd edition. New
York: Oxford University Press; 2001. p 242-243.
62
2009 AANEM Electrodiagnostic Self-Assessment Examination
Question 103: B
SUMMARY:
Sciatic neuropathy is uncommon and usually suggests a structural etiology. Examples include
complications of hip fracture or hip surgery, vascular anomalies and rarely fibromuscular bands; rarely the
nerve is entrapped by the piriformis muscle. Infarction of the femoral nerve is much more common than
the sciatic. Tight belts around the waist increase the risk of meralgia paresthetica, which results from
injury to the lateral femoral cutaneous nerve. Retroperitoneal hematoma may infiltrate and injure the
lumbosacral plexus.
REFERENCE:
Preston DC, Shapiro BE. Electromyography and neuromuscular disorders, 2nd edition. Philadelphia:
Elsevier; 2005. p 493.
Question 104: C
SUMMARY:
Posttetanic potentiation occurs after prolonged repetitive nerve stimulation or after prolonged voluntary
muscle contraction. It is caused by an increase in the immediately available stores of acetylcholine due to
an increase in mobilization rate. This change coupled with increased calcium accumulation in the nerve
terminal, enhances release of acetylcholine and subsequently an increase in the excitatory end-plate
potential which results in posttetanic potentiation for the following 1-2 minutes.
REFERENCE:
Kimura J. Electrodiagnosis in diseases of nerve and muscle: principles and practice, 3rd edition. New
York: Oxford University Press; 2001. p 252.
Question 105: D
SUMMARY:
Increasing the sensitivity during the recording of a compound muscle action potential (CMAP) will result in
an earlier observable deviation from the baseline for the onset latency. This will result in a decreased
visually measured latency to the onset of the CMAP. Many current machines set latency automatically at
a fixed gain setting irrespective of display settings.
REFERENCE:
Kimura, J, Electrodiagnosis in diseases of nerve and muscle: principles and practice, 3rd edition. New
York: Oxford University Press; 2001. p 98-100.
Question 106: B
SUMMARY:
After the age of 50 years there is a slowing on motor nerve conduction studies of 1 to 2 m/s with each
advancing decade. These changes are associated with a reduction in the motor amplitude, prolongation of
the distal motor latency, and an increase in the response duration.
REFERENCE:
Dumitru D, Amato AA, Zwarts MJ. Electrodiagnostic medicine, 2nd edition. Philadelphia: Hanley and
Belfus; 2002. p 187.
2009 AANEM Electrodiagnostic Self-Assessment Examination
63
Question 107: D
SUMMARY:
Nonhereditary amyloid neuropathy is predominantly sensory with prominent early loss of small fibers,
followed by progressive weakness and large-fiber involvement. Dysautonomia is often severe and
disabling, as is the pain associated with small fiber damage. Diagnosis depends on the histological
demonstration of amyloid either in the rectal or nerve biopsy. Rectal biopsy is positive in 70% of cases,
but it must include the submucosa because that tissue is involved more frequently than mucosa.
REFERENCES:
Dyck PJ, Thomas PK. Peripheral neuropathy, 4th edition. Philadelphia: Elsevier Saunders; 2005. p 2436.
Oh SJ. Principles of clinical electromyography: case studies. Baltimore: Williams & Wilkins; 1998. p 297.
Question 108: C
SUMMARY:
Decreasing the low frequency filter allows for more of the low frequency components of a motor action
potential to be recorded. This in turn will increase the compound muscle action potential (CMAP)
amplitude. Increasing the stimulus intensity above supra-maximal has no significant effect on the CMAP
amplitude. It does, however, increase the risk for volume conduction and depolarization of adjacent nerve
not being tested. Increasing extremity temperature will have no significant effect upon CMAP amplitude in
general. At significantly elevated temperature the CMAP amplitude will decrease. Increasing the distance
between the stimulating and recording electrodes will reduce the CMAP amplitude due to physiologic
temporal dispersion o the signal. Increasing the electrode impedance will reduce the amplitude of the
CMAP response by a reduction in the signal to noise ratio.
REFERENCE:
Kimura J. Electrodiagnosis in diseases of nerve and muscle: principles and practice, 3rd edition. New
York: Oxford University Press; 2001. p 109-110.
Question 109: E
SUMMARY:
End-plate activity recorded by needle electromyography has two forms, monophasic and biphasic.
Monophasic activity, also known as miniature end-plate potentials, are negative potentials and have a
characteristic “seashell sound.” Biphasic activity, also known as end-plate spikes, show negative-positive
spikes. Both types of activity are irregular and limited to localized areas of muscle. Typically, areas where
end-plate activity is found are more painful to the patient.
REFERENCE:
Dumitru D, Amato AA, Zwarts MJ. Electrodiagnostic medicine, 2nd edition. Philadelphia: Hanley & Belfus;
2002. p 265-266.
Question 110: D
SUMMARY:
A complaint of hand weakness and a finding of thumb flexion weakness raises the question of an anterior
interosseous neuropathy. This syndrome involves a lesion of the anterior interosseous nerve which
innervates the median component of the flexor digitorum profundus, flexor pollicis longus, and pronator
quadratus. Given the clinical weakness of the flexor pollicis longus, finding needle electromyography
evidence of denervation in any one of the other three muscles is supportive.
REFERENCE:
Dumitru D, Amato AA, Zwarts MJ. Electrodiagnostic medicine, 2nd edition. Philadelphia: Hanley & Belfus;
2002. p 1057-1058.
64
2009 AANEM Electrodiagnostic Self-Assessment Examination
Question 111: E
SUMMARY:
A shock artifact appears after nerve stimulation and enlarges with increasing current. An A wave, F wave,
and H reflex appear after the M response (which is present and decreases in latency with inching).
REFERENCE:
Dumitru D, Amato AA, Zwarts MJ. Electrodiagnostic medicine, 2nd edition. Philadelphia: Hanley & Belfus;
2002. p 93-94.
Question 112: D
SUMMARY:
Botulinum toxin (BT) acts presynaptically by interfering with the process of quantal release of
acetylcholine (ACh). This results in reduction of the end-plate potential and impairment of neuromuscular
transmission. BT does not interfere with the calcium channels, binding of ACh at the postsynaptic
receptor, or muscle membrane depolarization. How exactly BT impairs the ACh release is still not fully
known, but it disrupts the normal binding of synaptosomal vesicles to the axon terminal membrane.
Different toxin strains appear to affect different critical nerve terminal proteins. The toxins are zinc
proteases that cleave or inactivate various proteins necessary for ACh vesicle binding, fusion, and
release, e.g., synaptobrevin, vesicle-associated membrane protein, and syntaxin.
REFERENCE:
Dumitru D, Amato AA, Zwarts MJ. Electrodiagnostic medicine, 2nd edition. Philadelphia: Hanley & Belfus;
2002. p 479-482.
Question 113: E
SUMMARY:
Needle electromyography silence occurs with muscle contracture, myoedema, and an attack of
hypokalemic periodic paralysis. Clinical cramps associated with certain glycogenoses, including
myophosphorylase deficiency (McArdle disease) and phosphofructokinase deficiencies (Tauri disease),
are technically contractures, because of the electrical silence. Insertion of a needle electrode into a region
of muscle experiencing myoedema demonstrates electrical silence. Isaacs syndrome is a hyperexcitable
peripheral nerve disorder characterized by stiffness, continuous muscle contraction, and continuous motor
unit activity on needle EMG. Neuromyotonic discharges, myokymia, and other phenomena are associated
with the continuous muscle membrane activity in Isaacs syndrome, in some cases associated with
voltage-gated potassium channel antibodies.
REFERENCES:
Kimura J. Electrodiagnosis in diseases of nerve and muscle: principles and practice, 2nd edition.
Philadelphia: FA Davis; 1989. p 564, 569.
Preston DC, Shapiro BE. Electromyography and neuromuscular disorders: clinical electrophysiologic
correlations, 2nd edition. Philadelphia: Elsevier Butterworth-Heineman; 2005. p 211, 603.
Oh SJ. Principles of clinical electromyography: case studies. Baltimore: Williams & Wilkins; 1998. p 529.
2009 AANEM Electrodiagnostic Self-Assessment Examination
65
Question 114: B
SUMMARY:
Following nerve injury, the distal axons undergo Wallerian degeneration. Within hours, myelin begins to
retract from the axons at the nodes of Ranvier and form myelin ovoids, but the Schwann cells persist. By
the end of the first week, the axon and myelin become fully digested and Schwann cells start to bridge the
gap between the two nerve segments. Also, the neuronal cell body reacts to the axonal injury, by
revealing an eccentric nucleus and marginally placed rough endoplasmic reticulum (Nissl substance).
REFERENCE:
Katirji B. Electromyography in clinical practice: a case study approach, 2nd edition. Philadelphia: Elsevier;
2007. p 65.
Question 115: A
SUMMARY:
The earliest and most common electrophysiologic changes in acute inflammatory demyelinating
polyneuropathy is delayed, absent, or in persistent F-wave and H-reflex responses, usually encountered
within a few days and reflecting proximal demyelination. Delayed motor terminal latency, conduction
block, and temporal dispersion occur later and are present in about 50% of patients by 2 weeks and in
85% by 3 weeks. However, a small number of patients have inexcitable nerves early on, either from
axonal degeneration or from presumed distal demyelination.
REFERENCE:
Preston DC, Shapiro BE. Electromyography and neuromuscular disorders. Boston: ButterworthHeinemann; 1998. p 362–365.
Question 116: E
SUMMARY:
Lower temperatures slow impulse propagation while at the same time augmenting amplitude. Distal
latencies increase by 0.3 ms per degree Celsius (C) for both median and ulnar nerves upon cooling the
hand. Skin temperature should be maintained at 34º C or above to be consistent with normative data. One
may add 5 percent of the calculated conduction velocity for each degree below 32º C to normalize the
result, however this may provide misleading interpretation in diseases of the peripheral nerve.
REFERENCE:
Kimura, J. Electrodiagnosis in diseases of nerve and muscle: principles and practice, 3rd edition. New
York: Oxford University Press; 2001. p 109-110.
Question 117: D
SUMMARY:
Myotonic discharges are the repetitive firing of a single muscle fiber, typically at a rate of 20-100 Hz with a
waxing and waning firing pattern. Fasciculation potential, myokymic discharges, neuromyotonic
discharges, and cramp discharges are types of abnormal spontaneous activity generated by one or
multiple motor units.
REFERENCE:
Dumitru D, Amato AA, Zwarts MJ. Electrodiagnostic medicine, 2nd edition. Philadelphia: Hanley & Belfus;
2002. p. 277-278.
66
2009 AANEM Electrodiagnostic Self-Assessment Examination
Question 118: C
SUMMARY:
Kennedy syndrome, also known as bulbospinal muscular atrophy, is X-linked and is associated with
increased trinucleotide repeat sequence of cystine, arginine, and guanine in an androgen receptor on
Xq12. Dysferlin-associated myopathies, including limb girdle muscular dystrophy and distal Miyoshi
myopathy, are typically autosomal recessive. Charcot-Marie-Tooth (CMT) types 1A and 1B, linked to
defects in the peripheral myelin protein-22 (PMP-22) and myelin protein zero (MPZ) gene respectively,
are typically autosomal dominant and the most common forms of CMT. The third most common form of
CMT is an X-linked type associated with connexin-32 gene mutations, and is typically termed CMT-X, but
is not considered to be a form of primary demyelinating CMT or CMT1. Facioscapulohumeral dystrophy
and proximal myotonic myopathy, also known as PROMM or type 2 myotonic dystrophy, are typically
autosomal dominant.
REFERENCES:
Auer-Grumbach M, Wagner K, Strasser-Fuchs S, Loscher WN, Fazekas F, Milner M, Hartung HP. Clinical
predominance of proximal upper limb weakness in CMT1A syndrome. Muscle Nerve
2000;23:1243–1249.
Katirji B, Kaminski HJ, Preston DC, Ruff RL, Shapiro BE. Neuromuscular disorders in clinical practice.
Boston: Butterworth Heinemann; 2002. p 459
Preston DC, Shapiro BE. Electromyography and neuromuscular disorders, 2nd edition. Philadelphia:
Elsevier; 2005. p 453.
Question 119: C
SUMMARY:
The amplitude of the action potential is directly proportional to the distance between the generator of the
potential (nerve or muscle) and the recording electrode. With antidromic sensory technique (such as with
median sensory studies), the recording electrode is closer to the nerve (digits) than with an orthodromic
sensory technique (wrist). The proximity of the stimulator to the nerve plays no role since all stimulations
are supramaximal.
REFERENCE:
Preston DC, Shapiro BE. Electromyography and neuromuscular disorders, 2nd edition. Philadelphia:
Elsevier; 2005. p 30-32.
Question 120: B
SUMMARY:
Monopolar needle recordings reveal motor unit action potentials (MUAPs) with larger amplitudes and
more phases. The duration of the MUAP is believed to be the same with both monopolar and concentric
needle electrodes.
REFERENCE:
Dumitru D, Amato AA, Zwarts MJ. Electrodiagnostic medicine, 2nd edition. Philadelphia: Hanley & Belfus;
2002. p 308-310.
2009 AANEM Electrodiagnostic Self-Assessment Examination
67
Question 121: A
SUMMARY:
Miniature end-plate potentials are the extracellularly recorded potentials when the needle tip is close to
the end-plate region of the muscle. They represent non-propagating depolarizations caused by
spontaneous release of acetylcholine quanta. Nerve twig potentials are recorded as end-plate spikes,
sometimes in addition to end-plate noise. Ephaptic transmission among muscle fibers is the generator of
complex repetitive discharges.
REFERENCE:
Katirji B. Electromyography in clinical practice: a case study approach, 2nd edition. Philadelphia: Elsevier;
2007. p 26.
Question 122: D
SUMMARY:
As display sensitivity is increased, a smaller deflection from the baseline of the waveform can be
observed. This will result in a shortening of the visually determined onset latency. Increasing the display
sensitivity will not appreciably alter the visually measured amplitude or peak latency of the compound
muscle action potential.
REFERENCES:
Gitter A, Stolov WC. AAEM Minimonograph #16: Instrumentation and measurement in electrodiagnostic
medicine. Muscle Nerve 1995;18:816.
Dumitru D, Amato AA, Zwarts MJ. Electrodiagnostic medicine, 2nd edition. Philadelphia: Hanley & Belfus;
2002. p 553-554.
Question 123: B
SUMMARY:
This radiculopathy is recent and may be associated with decreased recruitment, the electrodiagnostic
counterpart of clinical weakness. The motor unit action potentials are normal because collateral sprouting
has not yet occurred. This process usually starts one month after injury. Complex repetitive discharges are
seen in more chronic neurogenic or myopathic conditions. Sensory responses should be spared in a
radiculoapthy unless a superimposed process is present. Myokymic discharges are usually seen after
radiation therapy and, rarely, with chronic radiculopathy.
REFERENCE:
Brown WF, Bolton CF. Clinical electromyography, 2nd edition. Boston: Butterworths; 1990. p 189-191.
68
2009 AANEM Electrodiagnostic Self-Assessment Examination
Question 124: D
SUMMARY:
Electrodiagnostic studies can show limited, but potentially useful, information immediately after nerve
injury, including baseline distal evoked amplitude, confirmation of the site of injury if severe or complete,
or support for psychogenic weakness over nerve injury in some cases. In this example disruption of distal
axons and surrounding elements due to neurotmesis would lead to an inability to elicit a significant
compound muscle action potential from axillary stimulation despite distal preservation of excitability.
Similarly, both neurapraxia and axonotmesis type injury would show preserved excitability below the
lesion site. The response would progressively decay over 5 to 7 days due to Wallerian degeneration in
axonotmesis and neurotmesis, but not neurapraxia. A similar response distally at day 10 post injury would
imply significant neurapraxia and a favorable prognosis for recovery. Fibrillation potentials would take 10
to 14 days to begin to appear.
REFERENCE:
Kimura J. Electrodiagnosis in disease of nerve and muscle, 3rd edition. New York: Oxford University
Press; 2001. p 99-103.
Question 125: B
SUMMARY:
The most common result of this error is a prolongation of latency (which results in slowing of conduction
velocity) because the nerve depolarizes under the cathode which is now placed further away from the
recording electrode. Since the distance between the cathode and anode in most stimulators is about 2.5
to 3.0 cm, the prolongation of latency will be about 0.3 to 0.4 ms in healthy nerves. The conduction
velocity will change as well because of the longer actual than measured distance. The amplitude may
minimally change because of the increased distance, but this effect is much less prominent than the effect
on latency. Another result of reversing the stimulating anode and cathode is anodal block which results
from hyperpolarization of axons under the anode, but this effect is much less notable than the latency and
resultant conduction velocity difference.
REFERENCE:
Preston DC, Shapiro BE. Electromyography and neuromuscular disorders, 2nd edition. Philadelphia:
Elsevier; 2005. p 94-95.
Question 126: D
SUMMARY:
Both afferent and efferent pathways of the heart rate variability induced by cyclic deep breathing are
transmitted by parasympathetic cholinergic fibers. The test is one of the best and most reproducible
parasympathetic measures. The heart rate, not blood pressure response to valsalva, is primarily
parasympathetic cholinergic, but has other complex influences. Both sympathetic skin responses (SSR)
and quantitative sudomotor axon reflex test sympathetic cholinergic efferent pathways. The SSR afferent
response is variably depending on the type of trigger, but is most commonly an electrical stimulus of
somatic sensory fibers. The cold pressor test is an older uncommonly used measure of sympathetic
function; the subjects arm is immersed in ice water and the rise in diastolic blood pressure is measured.
REFERENCE:
Low PA. Clinical autonomic disorders, 2nd edition. Philadelphia: Lippincott Raven; 1997. p 297-307.
2009 AANEM Electrodiagnostic Self-Assessment Examination
69
Question 127: A
SUMMARY:
Myokymic discharges are often seen on needle recording in patients who develop symptoms of brachial
plexopathy following a course of radiation therapy. The finding of myokymic discharges can aid in
distinguishing a radiation plexopathy from recurrent tumor. Myokymic discharges are not seen in the other
disease processes listed.
REFERENCE:
Dumitru D, Amato AA, Zwarts MJ. Electrodiagnostic medicine, 2nd edition. Philadelphia: Hanley & Belfus;
2002. p 821-822.
Question 128: B
SUMMARY:
A slow rate of repetitive stimulation depletes the immediate store of acetylcholine. In normal muscle, the
large margin of safety ensures that even reduced levels of acetylcholine are adequate to excite the
muscle fibers above threshold. The amount of acetylcholine released is unrelated to local
acetylcholinesterase levels or to the amplitude of the resulting motor response.
REFERENCE:
Kimura J. Electrodiagnosis in diseases of nerve and muscle: principles and practice, 3rd edition. New
York: Oxford University Press; 2001. p 263.
Question 129: A
SUMMARY:
The figure demonstrates Froment’s sign, which is diagnostic of ulnar nerve palsy caused by weakness of
the adductor pollicis and palmar interossei. There is associated compensation wherein the flexor pollicis
longus muscle is used to flex the distal interphalangeal joint of the thumb. Among long-distance cyclists,
ulnar neuropathy is called “handlebar palsy” and is caused by compression of the ulnar nerve at the hand
and wrist during a long-distance ride. Ulnar nerve palsy leads to weakness of the ulnar-innervated
muscles of the hand but not the lateral two lumbricals, opponens pollicis, abductor pollicis brevis, and
flexor pollicis brevis (superficial head) muscles, which are supplied by the median nerve.
REFERENCE:
Dumitru D, Amato AA, Zwarts MJ. Electrodiagnostic medicine, 2nd edition. Philadelphia: Hanley & Belfus;
2002. p 1075.
Question 130: D
SUMMARY:
The brief bursts of activity noted to correlate with eyelid blinking can be seen as a consequence of
aberrant axonal regeneration after nerve injury (synkinesis). In this case fibers that originally innervated
the orbicularis oculi now partially innervate other facial muscles.
REFERENCE:
Kimura, J. Electrodiagnosis in diseases of nerve and muscle: principles and practice, 3rd edition. New
York: Oxford University Press; 2001. p 422-423.
70
2009 AANEM Electrodiagnostic Self-Assessment Examination
Question 131: E
SUMMARY:
The sympathetic skin response is a long latency response with predominately very low frequency
components. This requires that the low frequency filter (high pass filter) be set very low to optimize the
recording of the low frequency component. All of the remaining potentials noted above are made up
primarily of high frequency components and would benefit from the low frequency filter being set at a
higher level.
REFERENCE:
Preston DC, Shapiro BE. Electromyography and neuromuscular disorders: clinical-electrophysiologic
correlations, 2nd edition. Philadelphia: Elsevier; 2005. p 115-117.
Question 132: B
SUMMARY:
During motor conduction studies, correct placement of the active electrode (G1) is crucial and should be
on the motor point of the muscle. This is usually located at the midpoint of the muscle halfway between its
insertion and origin. The compound muscle action potential (CMAP) is a biphasic potential with an initial
negativity, or upward deflection from the baseline, if the recording electrodes have been properly placed.
Failure to place G1 over the motor point will result in an initial positive deflection in the CMAP response,
i.e., triphasic, initial positive deflection waveform. Additionally, incorrect placement of G1 will result in
decreased CMAP amplitude.
REFERENCE:
Dumitru D, Amato AA, Zwarts M. Electrodiagnostic medicine, 2nd edition. Philadelphia: Hanely & Belfus;
2002. p 174-176.
Question 133: D
SUMMARY:
A motor unit action potential (MUAP) firing at 25 Hz in isolation is abnormally fast and implies a reduction
in the size of the available motor neuron pool. Large amplitude (MUAPs) with decreased recruitment
occur in post polio cases. The large amplitude MUAPs are the result of an increase in the size of the
motor unit and fiber type grouping, consequent on axonal sprouting from surviving motor neurons, during
reinnervation of denervated muscle fibers. Myopathic states such as myotonic dystrophy result in early
recruitment of small amplitude short duration polyphasic MUAPs; chronic polymyositis may produce some
larger amplitude, polyphasic motor unit potentials, but not in isolation. At the firing rate of 25 Hz, many
other motor units should be simultaneously firing. Neuromuscular junction disorders mimic the changes
seen with myopathy. Short duration low amplitude MUAPs may be observed in myasthenia gravis.
REFERENCE:
Dumitru D. Electrodiagnostic medicine. Philadelphia: Hanley & Belfus; 1995. p 481.
2009 AANEM Electrodiagnostic Self-Assessment Examination
71
Question 134: E
SUMMARY:
Assuming a conduction velocity of at least 50 m/s and signal duration of 0.8 ms, spacing the recording
electrodes 4 cm apart will permit recording the signal passing the G1 electrode before it is recorded at the
G2 electrode, resulting in less distortion from simultaneous recordings of the signal at G1 and G2. Many
labs utilize distances of 2 cm or 3 cm as there may be less noise and application of the electrodes to the
digits is easier. Normal values should reflect the precise methods employed in each lab.
REFERENCE:
Kimura J. Electrodiagnosis in diseases of nerve and muscle: principles and practice, 3rd edition. New
York: Oxford University Press; 2001. p 107.
Question 135: B
SUMMARY:
The pudendal nerve is the direct continuation of the lower band of sacral plexus. It exits the pelvis via the
greater sciatic foramen below the piriformis muscle to enter the gluteal region. The most common cause
of injury to the pudendal nerve or it branches is parturition. Damage occurs by mechanical disruption of
muscle fibers or disruption of the innervation to skeletal muscle. Bicycle riding, blunt trauma, and
penetrating injuries can injure the pudendal nerve, but not skate boarding.
REFERENCE:
Leis AA, Trapani VC. Atlas of electromyography. New York: Oxford University Press; 2000. p 158.
Question 136: E
SUMMARY:
A decrease in temperature slows down both opening and closing of the sodium channel with closing being
more affected than opening. This results in slower depolarization, hence a decreased conduction velocity.
The amplitude of the nerve action potentials is increased as the temperature is decreased. Because the
sodium channel stays open longer, more sodium ions flow into the cell during the depolarization and the
amplitude of the response increases.
REFERENCES:
Kimura J. Electrodiagnosis in diseases of nerve and muscle: principles and practice, 3rd edition. New
York: Oxford University Press; 2001. p 68.
Rutkove SB. AAEM Minimonograph #14: Effects of temperature on neuromuscular electrophysiology.
Muscle Nerve 2001;24:867-882.
Question 137: D
SUMMARY:
The American Association of Electrodiagnostic Medicine Guidelines for Ethical Behavior and the
American Medical Association Code of Medical Ethics call for technically sound, quality studies. Because
there is uncertainty about the presence or absence of carpal tunnel syndrome (CTS) in this patient, the
risk of making a false positive diagnosis of CTS (and possibly consigning the patient to surgery)
outweighs the inconvenience and lost revenue of the supervising electrodiagnostic medicine physician for
rescheduling and performing a technically adequate study. The possibility of “double crush” makes option
E an erroneous move. Too much is at stake for the patient to rely on inference here.
REFERENCES:
American Association of Electrodiagnostic Medicine. Guidelines for ethical behavior relating to clinical
American medical association council on ethical and judicial affairs. Code of medical ethics: current
opinions with annotations, 1998-1999 edition. Chicago: American Medical Association; 1998.
72
2009 AANEM Electrodiagnostic Self-Assessment Examination
Question 138: E
SUMMARY:
This clinical presentation is consistent with diabetic amyotrophy now often termed lumbosacral
radiculoplexoneuropathy, which usually occurs in elderly men with type II diabetes. Femoral nerve
infarction can produce a similar clinical picture but is restricted to femoral nerve function. Hemophilia is
rarely associated with retroperitoneal hematoma, and lumbar plexopathy with anterior thigh pain and
weakness. Uremia and alcoholism usually cause a symmetric distal peripheral polyneuropathy and, rarely,
a proximal myopathy. Multiple myeloma is associated with a distal polyneuropathy, which may be
demyelinating, axonal, or a mixed pattern with both motor and sensory involvement and neuropathic pain.
REFERENCE:
Katirji B, Kaminski HJ, Preston DC, Ruff RL, Shapiro EB. Neuromuscular disorders in clinical practice.
Boston: Butterworth-Heinemann; 2002. p 608–611.
Question 139: C
SUMMARY:
Temperature is one of the most important physiological variables in nerve conduction studies. With
cooling, the motor and sensory conduction velocities slow between 1.5 and 2.5 m/s from every 1 degree
centigrade drop in temperature (the distal latency prolongs by approximately 0.2 ms per degree). In this
example, warming the limb will results in a change of 12 m/s to 20 m/s in conduction velocity (i.e., the
velocity will change to a value of 48 to 56 m/s). Beware that this conversion only applies to normal and
healthy nerves and hence may not apply to diseased nerves. Warming the studied limb is the most
accepted procedure.
REFERENCE:
Preston DC, Shapiro BE. Electromyography and neuromuscular disorders, 2nd edition. Philadelphia:
Elsevier; 2005. p 87-89.
Question 140: E
SUMMARY:
The H reflex is a submaximal response that is fairly constant in latency, is elicitable from the tibial nerve in
adults, and usually requires a 0.5 to 1.0 ms stimulation duration. It is generated by activation of insertional
activity afferents.
REFERENCE:
Dumitru D, Amato AA, Zwarts M. Electrodiagnostic medicine, 2nd Edition. Philadelphia: Hanley & Belfus;
2002. p 244-246.
2009 AANEM Electrodiagnostic Self-Assessment Examination
73
Question 141: C
SUMMARY:
Classically in myotonic dystrophy, the distal weakness precedes the proximal weakness. It is not unusual
for a patient with myotonic dystrophy to present with steppage gait as the initial symptom, and for a
neuropathy to be suspected. Myotonia is defined as the sustained muscle contraction caused by
spontaneous repetitive depolarization of the muscle membrane. The best muscles for percussion
myotonia are the thenar and wrist extensor muscles. As the disease progresses, myotonia becomes
difficult to detect because of muscle wasting. Decremental responses with repetitive nerve stimulation are
not uncommon in the myotonic syndromes. Although decrements may be seen with slow repetitive
stimulation at 2 to 3 Hz, they are more common with faster frequencies (more than 10 Hz). Myotonic
discharges often have a configuration similar to positive sharp waves but with differing firing
characteristics, but the two can be confused. The other findings listed are not highly specific.
REFERENCES:
Oh SJ. Principles of clinical electromyography: case studies. Baltimore: Williams & Wilkins; 1998. p 435,
439.
Preston DC, Shapiro BE. Electromyography and neuromuscular disorders, 2nd edition. Philadelphia:
Elsevier Butterworth-Heineman; 2005. p 594-595.
Question 142: D
SUMMARY:
An electrodiagnostic (EDX) medicine physician should have sufficient clinical acumen to make a clinical
decision and perform an examination if those examinations are the most efficient next step in solving a
clinical problem. Unless the EDX physician is also asked to conduct a clinical consultation, concerns
outside of the purview of the examination should be directly conveyed to the referring physician. The
consultant should not make treatment or evaluation recommendations directly to the patient.
REFERENCE:
American Association of Electrodiagnostic Medicine. Guidelines for ethical behavior relating to clinical
practice issues in electrodiagnostic medicine. Muscle Nerve 1999;22:S43–S47.
Question 143: D
SUMMARY:
Acquired demyelinating polyneuropathy occurs in association with a number of conditions including
human immunodeficiency virus infection, anti myelin-associated glycoprotein antibody, diphtheria infection
and osteosclerotic myeloma. Sjögren’s syndrome typically causes sensory ataxic polyneuropathy, but not
demyelinating polyneuropathy.
REFERENCE:
Katirji B. Electromyography in clinical practice: a case study approach, 2nd edition. Philadelphia: Elsevier;
2007. p 197.
74
2009 AANEM Electrodiagnostic Self-Assessment Examination
Question 144: A
SUMMARY:
Critical illness myopathy, also known as acute quadriplegic myopathy, is an increasingly recognized
condition in critically ill patients; steroid treatment and paralyzing agents are important risk factors, but are
not essential to produce the condition. A variety of pathological findings are known, but loss of myosin
heavy chain staining and resultant interruption of the normal muscle banding pattern is most
characteristic. Some cases have coincident or preexisting neuropathy, but the findings are most
commonly a lesser features. Compound motor action potential (CMAP) amplitudes are notably reduced in
some cases with diffusely increased response duration. Because the underlying physiology also involves
impairment of fast sodium channel inactivation and resultant muscle fiber inexcitability, direct muscle
stimulation typically shows reduced or absent responses in contrast to acute neurogenic processes, such
as axonal Guillain-Barré syndrome, which have preserved direct muscle responses, despite low CMAP
amplitudes. Conduction velocity is relatively preserved. Sensory responses are often partially preserved
or normal, unless neuropathy is also present. Recruitment is variable, often because the critically ill
patients are either too weak or are not sufficiently conscious to cooperate. If patients are able to provide
voluntary effort, the pattern is often myopathic or non-specific. Some cases show spontaneous activity,
but many do not.
REFERENCE:
Hirano M, Weimer LH. Acute quadriplegic myopathy in Rowland LP, editor. Merritt’s textbook of
neurology, 11th edition. Baltimore: Williams & Wilkins; 2005. p 888-889.
Question 145: E
SUMMARY:
The deltoid and triceps are innervated by the axillary nerve and radial nerve, respectively. Both nerves
originate from the posterior cord.
REFERENCE:
Dumitru D, Amato AA, Zwarts MJ. Electrodiagnostic medicine, 2nd edition. Philadelphia: Hanley & Belfus;
2002. p 780.
Question 146: D
SUMMARY:
Sympathetic skin responses can be reproducibly triggered following a noxious electrical stimulus at any
anatomical site. The course of the reflex is quite long and complex with afferent, central, and efferent
components. Responses from the palm of the hand referenced to the back of the hand have onset
latencies in the range of 1.5 sec or 1,500 ms. Despite the fact that much of the efferent pathways are
unmyelinated, the bulk of the delay is produced by central pathways and complex processing.
REFERENCE:
Low PA. Clinical autonomic disorders, 2nd edition. Philadelphia: Lippincott Raven; 1997. p 221-232.
Question 147: A
SUMMARY:
This presentation is most likely indicative of dermatomyositis. In this condition, motor unit action potentials
(MUAPs) are small (due to loss of muscle fibers). Increased numbers of MUAPs are recruited in order to
generate a given force.
REFERENCE:
Katirji B, Kaminski HJ, Preston DC, Ruff RL, Shapiro BE. Neuromuscular disorders in clinical practice.
Boston: Butterworth-Heinemann; 2002. p 1169-1175.
2009 AANEM Electrodiagnostic Self-Assessment Examination
75
Question 148: C
SUMMARY:
Lowering temperature results in a higher amplitude and longer duration of the sensory nerve action
potential (SNAP). Decreasing the distance between the active and reference electrodes to 2 cm or less
results in a lower amplitude of the SNAPs since depolarization may occur under both electrodes
simultaneously. The other answers have no effect on the SNAP amplitude.
REFERENCE:
Dumitru D, Amato AA, Zwarts MJ. Electrodiagnostic Medicine, 2nd Edition. Philadelphia: Hanley & Belfus;
2002. p 72-73, 188-189.
Question 149: D
SUMMARY:
D is the only appropriate choice because the physician who has performed the electrodiagnostic medicine
referral has an obligation to complete the care which is appropriate and is related to the referral. In this
case, the physician must at least offer to have direct contact with the patient and re-examine him before
being able to know whether non-narcotic analgesic medication would be appropriate.
REFERENCE:
American medical association council on ethical and judicial affairs. Code of medical ethics: current
opinions with annotations, 1998-1999 edition. Chicago: American Medical Association; 1998. p
141.
Question 150: E
SUMMARY:
An important temperature effect occurs in relation to electrodiagnostic evaluation of possible
neuromuscular junction diseases. A patient with a neuromuscular junction disorder and a cool limb can
have a normal repetitive stimulation study, resulting in a false negative.
REFERENCE:
Dumitru D, Amato AA, Zwarts MJ. Electrodiagnostic medicine, 2nd edition. Philadelphia: Hanley & Belfus;
2002. p 559-560.
76
2009 AANEM Electrodiagnostic Self-Assessment Examination